Past Exam Errors 2 Flashcards

1
Q

A five-year-old is investigated with a fasting study for suspected hypoglycaemia.
Which combination of plasma glucose, insulin, beta hydroxybutyrate (β-OH-butyrate), lactate and cortisol can
best be described as a normal metabolic response to fasting after 18 hours in a five-year-old?
The non-fasting normal range values are shown in [ ] brackets.

Plasma glucose
[3.5-8.0 mmol/L]
Insulin
[15-120 pmol/L]
β-OH-butyrate
[0.1-0.3 mmol/L]
Lactate
[0.3-2.0 mmol/L]
Cortisol
[150-450 mmol/L]
A. 3.1 10 1.5 0.5 500
B. 3.1 10 0.3 0.5 420
C. 3.3 15 1.4 3.8 450
D. 2.4 10 2.1 1.0 650
E. 2.9 15 1.8 1.0 90
A

A - BGL 3.1 insulin 10 ketones 1.5 lactate 0.5 cortisol 500

How well did you know this?
1
Not at all
2
3
4
5
Perfectly
2
Q

Newborn screening programs around the world carry out programs reflecting local prevalence and need.
Which one of the following congenital conditions has the highest prevalence in Australia and New Zealand?

A. Bilateral hearing impairment.
B. Cystic fibrosis.
C. Galactosaemia.
D. Hypothyroidism.
E. Neuroblastoma.
A

A - bilateral hearing impairment. 1/1,000.

CF 1/2,500
Galactosemia 1/50,000
Hypothyroidism (congenital) 1/3,000
Neuroblastoma 1/100,000

How well did you know this?
1
Not at all
2
3
4
5
Perfectly
3
Q

Which one of the following types of mutation is most likely to lead to the introduction of a premature stop
codon?
A. 1 base pair insertion in an exon.
B. 1 base pair insertion in the promoter.
C. 2 base pair duplication in an intron.
D. 3 base pair deletion in an exon.
E. 3 base pair inversion in an exon.

A

A - 1 base pair insertion in an exon. Need to shift reading frame.

Stop codon would need to be in coding region i.e. exon, not intron. Deletion/changes to 3 base pairs maintains reading frame and is less likely.

How well did you know this?
1
Not at all
2
3
4
5
Perfectly
4
Q

Dendritic cells are distinguished by an exceptional ability to carry out which one of the following immune
functions?

A. Antigen presentation.
B. Chemokine secretion.
C. Cytokine secretion.
D. Immunoglobulin secretion.
E. Phagocytosis.
A

A - APC

How well did you know this?
1
Not at all
2
3
4
5
Perfectly
5
Q
Which one of the following poses the greatest threat to validity in a longitudinal cohort study?
A. Confounding.
B. Data collection intervals too long.
C. Data collection intervals too short.
D. Loss to follow-up.
E. Selection bias.
A

D - loss to follow up

How well did you know this?
1
Not at all
2
3
4
5
Perfectly
6
Q

The use of artificial surfactant has been shown to reduce the incidence of which one of the following
complications of prematurity?

A. Chronic lung disease.
B. Periventricular haemorrhage.
C. Pneumothorax.
D. Retinopathy of prematurity.
E. Symptomatic patent ductus arteriosus.
A

C - pneumothorax

How well did you know this?
1
Not at all
2
3
4
5
Perfectly
7
Q

The correlation between television violence and aggressive behaviour has been shown to be highest for
which one of the following age groups?

A. Pre-school.
B. Primary school.
C. Early adolescence.
D. Late adolescence.
E. Young adults.
A

A - pre-school

How well did you know this?
1
Not at all
2
3
4
5
Perfectly
8
Q

Which one of the following chemotherapeutic agents has activity in the S phase of the cell cycle only?

A. Cisplatin.
B. Etoposide (VP-16).
C. Ifosfamide.
D. Prednisolone.
E. Vinblastine.
A

D - prednisolone

Antimetabolites would be the more common/logical answer? None of the agents listed are antimetabolites.

Some antimetabolites that are commonly used to treat cancer include:
6-mercaptopurine.
fludarabine.
5-fluorouracil.
gemcitabine.
cytarabine.
pemetrexed.
methotrexate.

Others

Cisplatin is an alkylating agent, and works independently of the cell cycle.

Ifosfamide is cell phase nonspecific as well.

Etoposide’s mechanism of action is to cause single-strand and double-strand breaks in DNA through interaction with DNA topoisomerase II, inducing arrest in the G2-phase of the cell cycle.

Vinblastine is also an alkylating agent, and is cell cycle phase specific; it binds to microtubular proteins in the mitotic spindle, thereby preventing cell division during metaphase.

How well did you know this?
1
Not at all
2
3
4
5
Perfectly
9
Q
If one identical twin develops schizophrenia, what is the lifetime risk that the other twin will also develop
schizophrenia?
A. 25%.
B. 50%.
C. 75%.
D. 90%.
E. >99%.
A

B - 50%

How well did you know this?
1
Not at all
2
3
4
5
Perfectly
10
Q

You are providing safety advice to the parents of a six-year-old child of average height and weight.
Which one of the following is the most appropriate car seating arrangement for this child, in order to
decrease the chance of injury in a motor vehicle accident?

A. Back seat; booster; seat belt.
B. Back seat; no booster; seat belt.
C. Front or back seat; booster; seat belt.
D. Front seat; booster; seat belt.
E. Front seat; no booster; seat belt.
A

A - back seat, booster, seat belt.

Vicroads:

Children under 4 years old must travel in a rear facing or forward facing child restraint.

Children aged between 4-7 years must travel in a forward facing child restraint or a booster seat.

Children aged between 7 and 16 are required to use a booster seat or adult seat belt when travelling in a vehicle.

NSW:

National child restraint laws

  • Children up to the age of six months must be secured in an approved rearward facing restraint
  • Children aged from six months old but under four years old must be secured in either a rear or forward facing approved child restraint with an inbuilt harness
  • Children under four years old cannot travel in the front seat of a vehicle with two or more rows
  • Children aged from four years old but under seven years old must be secured in a forward facing approved child restraint with an inbuilt harness or an approved booster seat
  • Children aged from four years old but under seven years old cannot travel in the front seat of a vehicle with two or more rows, unless all other back seats are occupied by children younger than seven years in an approved child restraint or booster seat
  • Children aged from seven years old but under 16 years old who are too small to be restrained by a seatbelt properly adjusted and fastened are strongly recommended to use either a forward-facing seat with an in-built harness for older children, an approved booster seat, or an approved child safety harness in conjunction with the vehicle’s seatbelt
  • Children in booster seats must be restrained by a suitable lap and sash type approved seatbelt that is properly adjusted and fastened, or by a suitable approved child safety harness that is properly adjusted and fastened.
How well did you know this?
1
Not at all
2
3
4
5
Perfectly
11
Q

The diagram below shows cumulative quantal dose-response curves for a hypothetical drug (drug X). The
Y-axis represents the percentage of the population studied who: for curve (a) achieved the desired
therapeutic effect at a given dose; and for curve (b) experienced the most important toxic effect at a given
dose.

Based on this information, the best estimate of the therapeutic index of drug X is approximately:
A. 0
B. 0.3
C. 0.7
D. 1.0
E. 3.0
A

E - 3.0

TI = Toxic50/Therapeutic50

Toxic50 = dose at which 50% have toxic effect
Therapeutic 50 = dose at which 50% have therapeutic effect

How well did you know this?
1
Not at all
2
3
4
5
Perfectly
12
Q

The impact of an intervention in clinical trials and in systematic reviews can be expressed in a number of
ways. One increasingly used format is the number needed to treat (NNT) which indicates how many patients have to be treated with the intervention of interest compared to the control intervention in order to achieve one successful outcome.
In a systematic review of optimal home-management for asthma, the intervention was found to produce a
50% reduction in hospitalisation for asthma. Approximately 10% of patients in the control group required hospitalisation compared to approximately 5% of those who received optimal home-management.

Which one of the following is the best estimate of the NNT for this intervention?
A. 2.
B. 5.
C. 10.
D. 20.
E. 50.
A

D - 20

When a study outcome is expressed as a percent, the number needed to treat (NNT) is the inverse of the absolute risk reduction (ARR) expressed as a decimal. The example below compares an event rate of 26% versus 16%:

ARR=(Control event rate)−(Experimental event rate)
ARR=0.26−0.16=0.1
NNT=1/ARR
NNT=1/0.1 = 10patients

In the question, 0.1-0.05 = 0.05, 1/0.05 = 20

How well did you know this?
1
Not at all
2
3
4
5
Perfectly
13
Q

A previously well term infant develops multifocal clonic seizures at 72 hours of age. Examination reveals a
bulging fontanelle. A cranial ultrasound shows a large intraventricular haemorrhage. Investigation of the
clotting profile shows:

platelet count 200 x 109/L [150-400]
prothrombin time (PT) 100 seconds [10-24]
activated partial thromboplastin time (APTT) 35 seconds [28-79]

The most likely diagnosis is:
A. haemorrhagic disease of the newborn.
B. factor VII deficiency.
C. factor VIII deficiency.
D. factor XII deficiency.
E. von Willebrand disease.
A

B - factor VII deficiency

HDN = vitamin K deficiency = affects factors 2, 7, 9, 10 = should affect both PT and APTT.

How well did you know this?
1
Not at all
2
3
4
5
Perfectly
14
Q
In the human foetus near term, the ratio of the output of the right ventricle compared to the left ventricle is
closest to:
A. 1:2
B. 1:1.5
C. 1:1
D. 2:1
E. 4:1
A

D - 2:1

How well did you know this?
1
Not at all
2
3
4
5
Perfectly
15
Q

Which one of the following pharmacokinetic parameters is most important in the optimal prescribing of beta-
lactam antibiotics?

A. The peak concentration/minimum inhibitory concentration (MIC) ratio.
B. Area under the plasma concentration versus time curve (AUC).
C. The AUC/MIC ratio.
D. Time above the MIC.
E. Both AUC/MIC ratio and peak concentration/MIC ratio.

A

D - time above the MIC

Alison’s notes - patterns of antimicrobial activity

Type I - Concentration-dependent killing
• As concentration of an antibiotic increases, rate of killing increases (best when peak 10x MIC)
• Eliminate bacteria more rapidly when concentrations significantly above MIC
• Significant post-antibiotic effect with duration dependent on peak
• Peak concentration limited by toxicity
Examples: Aminoglycosides, Fluoroquinolones
Goal of therapy: maximise concentrations (peak/MIC)

Type II - Time-dependent killing
• Mainly dependent on time at binding site to kill organisms
• Increasing concentration will not increase effectiveness (maximum killing rate at 2-4x MIC)
• Amount of time above MIC in any one dosing interval is best predictor of clinical response (dosing interval important)
• Minimal to NO post-antibiotic effect
Examples: Penicillins, Cephalosporins, Carbapenems, Erythromycin, Linezolid
Goal: maximise duration of exposure (time above MIC)

Type III - Time and concentration dependent
• Combination of the two
• Rate of bacterial killing related to both time above MIC and total exposure of antibiotic to organism
Examples: Azithromycin, Vancomycin, Clindamycin, Tetracyclines
Goal: maximise amount of drug (24hr AUC/MIC)

How well did you know this?
1
Not at all
2
3
4
5
Perfectly
16
Q
Anorexia nervosa is accompanied by significant hormonal changes. Which one of the following is least likely to be found in an adolescent girl with anorexia nervosa?
A. Increased plasma cortisol.
B. Increased plasma growth hormone.
C. Increased plasma oestradiol.
D. Normal plasma prolactin.
E. Normal plasma thyroxine (T4).
A

D - normal plasma prolactin. ?Outdated… there seems to be an implication that this was theorised/expected but hasn’t been proven.

Google searches:

  • The role of prolactin in anorexia nervosa is controversial and both hyperprolactinaemia and normoprolactinaemia were reported in patients with anorexia nervosa.
  • Theoretically, one would expect to find increases in prolactin secretion in patients with anorexia nervosa: the evidence for this is reviewed. However, relevant work to date seems to indicate that this is not so. These paradoxical findings are discussed, and areas of further research are outlined.

Uptodate - endocrine complications of AN.

Reproductive — Suppression of the hypothalamic-pituitary-ovarian axis results in hypogonadotropic hypogonadism with amenorrhea, estradiol deficiency, and infertility. Low LH, low oestradiol, low testosterone.

Adrenal — Increased hypothalamic-pituitary-adrenal (HPA) activity in the setting of the stress of chronic starvation leads to hypercortisolemia.

Growth hormone — Although growth hormone (GH) levels are high in AN, levels of the downstream hormone insulin-like growth factor-1 (IGF-1) are low, indicating a state of “resistance” to GH due to chronic starvation.

Thyroid — Women with AN often have a “euthyroid-sick” pattern of thyroid function tests due to chronic undernutrition: triiodothyronine (T3) levels are low, and reverse T3 (rT3) levels and the ratio of thyroxine (T4) to T3 are high. Thyroid-stimulating hormone (TSH) and T4 levels may be normal or low.

●Basal levels of oxytocin, a hormone that is involved in a range of physiologic processes including social behaviors, modulation of anxiety and depressive symptoms, energy homeostasis, and bone metabolism, are decreased.

How well did you know this?
1
Not at all
2
3
4
5
Perfectly
17
Q

A five-year-old boy was successfully treated for a testicular germ cell tumour at two years of age. Therapy
consisted of cisplatin, etoposide (VP-16) and bleomycin.

The most likely long-term complication of the
therapy is:
A. high tone hearing deficit.
B. pulmonary fibrosis.
C. renal failure.
D. secondary leukaemia.
E. sterility.
A

A - high tone hearing deficit. In general:

  • platinum compounds a/w ototoxicity and nephrotoxicity
  • etoposide
  • bleomycin: pulmonary fibrosis

Uptodate

One study of cisplatin ototoxicity: At a median 51 months following chemotherapy, hearing loss was found in 80 percent of the survivors, and 18 percent had severe or profound hearing loss. Tinnitus (which was reported by 40 percent of survivors) was also significantly correlated with reduced hearing at each frequency.

Other complications:

Cisplatin: The range of cisplatin-induced neurotoxicity includes peripheral neuropathy, ototoxicity (hearing impairment and tinnitus), vestibulopathy, and encephalopathy; the most common are peripheral neuropathy and ototoxicity.

Etoposide — Etoposide (VP-16) is a topoisomerase II inhibitor that is used extensively in the treatment of lung cancer, germ cell tumors, and refractory lymphomas. Although neurotoxicity is uncommon, even in high doses, peripheral neuropathy (less than 2 percent), mild disorientation, seizures, transient cortical blindness, and optic neuritis have been reported.

Bleomycin (lung injury): Rates of any grade of pulmonary toxicity range from 5 to 16 percent, and rates of fatal pulmonary toxicity have been in the range of 0 to 1 (for three courses) and 0 to 3 percent (for four courses). However, more recent data suggest that the overall risk of clinically-apparent and fatal bleomycin-induced lung injury may be lower.

How well did you know this?
1
Not at all
2
3
4
5
Perfectly
18
Q
Which one of the following is least likely to present with acute flaccid paralysis?
A. Bacterial meningitis.
B. Enterovirus 71 meningoencephalitis.
C. Guillain-Barré syndrome.
D. Infant botulism.
E. Poliomyelitis.
A

A - meningitis. Acute flaccid paralysis is a feature of all the others.

●Encephalitis – Encephalitis is defined as inflammation of the brain parenchyma and is manifested by signs of neurologic dysfunction. Characteristic clinical features include altered mental status (decreased level of consciousness, lethargy, personality change, unusual behavior) lasting at least 24 hours, seizures, and/or focal neurologic signs (eg, cranial nerve palsies, abnormal movements, weakness; 50 to 60 percent), often accompanied by fever, headache, nausea, and vomiting.

●Meningitis – Meningitis is inflammation of the meninges and is typically manifested by fever, headache, nausea, vomiting, photophobia, and stiff neck.

GBS: sudden onset of weakness, usually affecting lower limb, ascending paralysis, symmetrical weakness, pain a prominent feature, 50% sensory involvement.

Infant botulism: p/w floppy infant with poor feeding, weak cry, hypotonia, constipation. General botulism: symmetric descending flaccid paralysis, bulbar musculature initially.

Poliomyelitis: Poliovirus is a species of human enterovirus. A small fraction of patients with poliovirus infection develop central nervous system (CNS) infection -> death of motor neuron -> paralysis. Weakness may vary from one muscle or group of muscles, to quadriplegia, and respiratory failure. Tone is reduced, nearly always in an asymmetric manner. Proximal muscles usually are affected more than distal ones, and legs more commonly than arms. Reflexes are decreased or absent. The sensory examination is normal.

How well did you know this?
1
Not at all
2
3
4
5
Perfectly
19
Q
The chronic administration of a thiazide diuretic is associated with all of the following except:
A. hypercalciuria.
B. hyperglycaemia.
C. hyperuricaemia.
D. hypochloraemic alkalosis.
E. normal serum magnesium.
A

A - hypercalciuria. Thiazides block Na/Cl cotransported in the DCT.

Thiazides are a potential treatment for hypercalciuria. The overall affect in the kidney in response to diuretics is the increase reabsorption of sodium, and therefore calcium, lowering calcium levels in urine.

Uptodate

Most of the filtered calcium is reabsorbed, an effect that occurs throughout the nephron. Calcium reabsorption in the proximal tubule can be affected by the patient’s volume status as enhanced proximal sodium and water reabsorption due to volume depletion leads to increased passive proximal calcium reabsorption.

Proposed mechanisms — Two main mechanisms have been proposed to explain the effect of thiazides on calcium excretion, but their relative importance is uncertain:
●Increased proximal sodium and water reabsorption due to volume depletion, which leads to increased passive proximal calcium reabsorption
●Increased distal calcium reabsorption at the thiazide-sensitive site in the distal tubule and connecting segment

Diuretic therapy has variable effects on urinary calcium excretion according to the site of action:
●Calcium excretion is increased by osmotic diuretics (such as mannitol) and the carbonic anhydrase inhibitor acetazolamide.
●Calcium excretion is increased by loop diuretics due to diminished reabsorption. This effect may be beneficial in selected patients with hypercalcemia, but can be deleterious in neonates, leading to the development of nephrocalcinosis.
●Calcium excretion is diminished by thiazide-type diuretics by as much as 50 to 150 mg (1.3 to 3.8 mmol) per day due to increased reabsorption.

How well did you know this?
1
Not at all
2
3
4
5
Perfectly
20
Q
Which one of the following capsular polysaccharides of Neisseria meningitidis is least immunogenic?
A. A.
B. B.
C. C.
D. W-135.
E. Y.
A

B - B

How well did you know this?
1
Not at all
2
3
4
5
Perfectly
21
Q

Which one of the following children is least likely to have language skills in the normal range at age five
years?

A. 14 months old; conductive hearing loss (pure tone average 30db hearing loss bilaterally) at
successive hearing tests aged 10 and 13 months; no words.
B. 18 months old; no words.
C. 24 months old; 40 single words; no two-word combinations.
D. 30 months old; acquired severe hearing loss due to meningitis aged 24 months; normal intelligence.
E. 36 months old; many words; not using grammatical markers of tense and person.

A

E - 36 months old, many words, not using grammatical markers of tense or person.

How well did you know this?
1
Not at all
2
3
4
5
Perfectly
22
Q

Which one of the following is the major cause of physiologic neonatal jaundice?
A. Decreased bilirubin uptake by the hepatocyte.
B. Decreased hepatic bilirubin conjugation.
C. Decreased hepatic excretion of bilirubin.
D. Increased de novo bilirubin synthesis.
E. Increased enterohepatic circulation.

A

D - increased de novo bilirubin synthesis

How well did you know this?
1
Not at all
2
3
4
5
Perfectly
23
Q

In X-linked severe combined immunodeficiency (SCID), the affected gene codes for the common cytokine
receptor gamma chain whose principal function is:
A. nuclear regulation in T and B cells.
B. rearrangement of the T and B cell receptors.
C. signal transduction in T and B cell receptors.
D. T cell-induced immunoglobulin isotype switching.
E. transduction of gamma C-chain-dependent intracellular signals.

A

C - signal transduction in T and B cell receptors

How well did you know this?
1
Not at all
2
3
4
5
Perfectly
24
Q
Cisapride can cause prolonged Q-T interval and cardiac arrhythmia when used in combination with other
drugs. Which one of the following drugs would be most likely to cause arrhythmias in a patient taking
cisapride?
A. Amoxycillin.
B. Cefaclor.
C. Erythromycin.
D. Metronidazole.
E. Trimethoprim-sulfamethoxazole.
A

C - erythromycin. Note TMP/SMX listed in Parks as a cause, not in the table on uptodate.

MRCPCH - acquired causes of QT prolongation

Antibiotics: erythromycin, clarithromycin, azithromycin, TMP/SMX
Antifungals: azoles (flucon, itracon, ketocon)
Antihistamines: terfenadine
Antidepressants: TCAs
Antipsychotics: haloperidol, resperidone, chlorpromazine
Antiarrhythmic agents
Diuretics (due to K loss): furosemide

Electolyte disturbances: hypokalaemia, hypocalcaemia, hypomagnesaemia

Underlying medical conditions: complete AV block, severe bradycardia, sick sinus syndrome, myocardial dysfunction, CHF, myocarditis, endocrinopathies (hyperparathyroid, hypothyroid, phaechromocytoma), neurologic (encephalitis, head trauma, stroke, SAH), nutritional (alcohol, anorexia, starvation)

How well did you know this?
1
Not at all
2
3
4
5
Perfectly
25
Q

One of two siblings is diagnosed with type 1 (insulin-dependent) diabetes mellitus (IDDM). There is no family
history of IDDM. What is the approximate risk that the other sibling will develop IDDM?
A. 1%.
B. 6%.
C. 12%.
D. 25%.
E. 50%.

A

B - 6%

How well did you know this?
1
Not at all
2
3
4
5
Perfectly
26
Q

A Southern blot on tumour tissue from a patient with Burkitt lymphoma is shown above. The signal in the
tumour tissue indicates which one of the following?
A. Amplified DNA.
B. Amplified messenger RNA.
C. Amplified protein.
D. Increased half-life of DNA.
E. Increased half-life of messenger RNA.

A

A - amplified DNA

Southern blot = DNA
Northern = RNA
Western = protein

Southern was first (persons name), then the others (opposite Southern second, then Western). Think DNA->RNA->protein.

How well did you know this?
1
Not at all
2
3
4
5
Perfectly
27
Q
The electrocardiogram (ECG) shown above was obtained from a three-year-old girl with a systolic murmur.
Which one of the following is the most likely diagnosis?

(superior axis)

A. Perimembranous ventricular septal defect.
B. Persistent ductus arteriosus.
C. Primum atrial septal defect.
D. Pulmonary valve stenosis.
E. Subaortic stenosis.
A

C - primum ASD

ECG findings

ASD: Primum = superior axis, Secundum = RBBB and 1st degree block

TA has superior axis as well, but cyanotic lesion.

How well did you know this?
1
Not at all
2
3
4
5
Perfectly
28
Q

In general, compared with autosomal dominant disorders, autosomal recessive disorders are:

A. more severe more variable within families more likely to be due to new mutations
B. more severe more variable within families less likely to be due to new mutations
C. more severe less variable within families less likely to be due to new mutations
D. less severe more variable within families more likely to be due to new mutations
E. less severe less variable within families less likely to be due to new mutations

A

C - more severe, less variable within families, less likely to be due to new mutations

How well did you know this?
1
Not at all
2
3
4
5
Perfectly
29
Q
Which one of the following is the least important cause of impaired gas exchange in meconium aspiration
syndrome?
A. Alveolar oedema.
B. Bacterial infection.
C. Bronchiolar obstruction.
D. Pulmonary hypertension.
E. Surfactant dysfunction.
A

B - bacterial infection. Causes chemical pneumonitis, as intrauterine environment sterile.

MRCPCH - MAS

RFs: term or post term, SGA, perinatal asphyxia (fetal distress).
Rare in preterms

Major effects on lung function:

  • Airway blockage: increased airway resistence with ball-valve mechanism and gas trapping, high risk of pneumothorax
  • Chemical pneumonitis
  • Increased risk of infection: E. coli is most common
  • Surfactant deficiency: lipid content of meconium displaces surfactant from alveolar surface, PPHN

CXR changes begin as patchy infiltration and hyperinflation. A more homogenous opacification of the lung fields develops of 48 hours as chemical pneumonitis becomes a problem.

No evidence for routine suction if baby is vigorous and otherwise in good condition.

How well did you know this?
1
Not at all
2
3
4
5
Perfectly
30
Q

A 900 g, 26-week gestation neonate receiving neonatal-intensive care develops systemic sepsis on day
seven. Klebsiella pneumoniae is grown from the blood cultures with the following sensitivities:
ampicillin resistant
cefotaxime resistant
imipenem sensitive
gentamicin resistant
amikacin sensitive
The most likely mode of resistance of this organism is:
A. altered binding proteins.
B. chromosomally-mediated resistance.
C. extended spectrum beta lactamase production.
D. inducible beta lactamase production.
E. intrinsic resistance.

A

C - extended spectrum beta lactamase

Google searching:

ESBL producing bacteria, in order of frequency:

  • E. coli (67%)
  • Klebsiella pneumoniae (25%)
  • Pseudomonas, acinetobacter, enterobacter (8-9% together)
How well did you know this?
1
Not at all
2
3
4
5
Perfectly
31
Q
The following pharmacokinetic data are provided for five drugs.
Drug Volume of distribution (L) Plasma half-life (hr)
A 1190 21
B 4620 600
C 70 22
D 7 6
E 126 11
Which of the drugs is most likely to have the slowest rate of clearance from plasma?
A. Drug A.
B. Drug B.
C. Drug C.
D. Drug D.
E. Drug E.
A

D - drug D with L=7 and T1/2=6

Alison’s notes:

Clearance=Vd/T0.5

How well did you know this?
1
Not at all
2
3
4
5
Perfectly
32
Q

A 16-year-old girl with schizophrenia has been commenced on risperidone.
Which one of the following is the least likely side-effect?
A. Increased dream activity.
B. Menstrual irregularities.
C. Muscle stiffness.
D. Somnolence.
E. Weight loss.

A

E - weight loss

Weight gain is significant side effect of atypical antipsychotics.
EPSE are the key SE of typical antipsychotics.

How well did you know this?
1
Not at all
2
3
4
5
Perfectly
33
Q

T lymphocytes expressing both CD4 and CD8 on the cell surface are most likely to be found in which one of
the following anatomical locations in healthy individuals?
A. Bone marrow.
B. Lymph nodes.
C. Peripheral blood.
D. Spleen.
E. Thymus.

A

E - thymus

The early T cell progenitor (first step in differentiation) migrates from the bone marrow to the thymus. The rest of the T cell development occurs in the thymus, including double negative and double positive stages (referring to CD4 and CD8).

How well did you know this?
1
Not at all
2
3
4
5
Perfectly
34
Q
Which one of the following cardiac conditions is most commonly associated with neurofibromatosis?
A. Aortic root dilatation.
B. Cardiac rhabdomyomata.
C. Coarctation of the aorta.
D. Pulmonary stenosis.
E. Supravalvular aortic stenosis.
A

D - pulmonary stenosis

Cardiology manifestations of NF1 are pulmonary stenosis and coarctation.
Other causes of PS: Noonans (valvular), Williams (peripheral), DiGeorge, Alagille (peripheral), congenital rubella (peripheral).

NF1:

  • chromosome 17, 50% sporadic, variable expression
  • features:>6 CALM, >2 lisch nodules, auxillary freckling, optic glioma, >2 neurofibroma / 1 plexiform neurofibroma, osseous lesion, first degree relative.
How well did you know this?
1
Not at all
2
3
4
5
Perfectly
35
Q
For males entering puberty at an average age and height, the increase in height (cm) from the onset of
puberty to final height is closest to:
A. 17.
B. 21.
C. 25.
D. 29.
E. 33.
A

D - 29cm

How well did you know this?
1
Not at all
2
3
4
5
Perfectly
36
Q

Which immunological mechanism is the major cause of the destruction of the small intestinal villous
architecture in coeliac disease?
A. Activation of B lymphocytes.
B. Activation of T lymphocytes.
C. Secretion of anti-endomysial antibodies.
D. Secretion of antigliadin antibodies.
E. Secretion of tissue transglutaminase antibodies.

A

B - activation of T lymphocytes

How well did you know this?
1
Not at all
2
3
4
5
Perfectly
37
Q

A nine-year-old girl presents with obesity. Her height is on the 10th percentile, weight on the 97th
percentile and she has a round face. Shortening of the fourth metacarpals bilaterally are found and
confirmed by X-ray.
Which finding would be most helpful in establishing a diagnosis of pseudohypoparathyroidism?
A. Basal ganglia calcification.
B. Increased fasting plasma insulin/glucose ratio.
C. Increased plasma calcium.
D. Increased serum alkaline phosphatase.
E. Increased serum parathyroid hormone.

A

E - increased serum PTH

MRCPCH

Autosomal dominant. End organ resistance to raised levels of PTH.
Abnormal phenotype of short stature, obesity, ID, round face, short neck, shortened 4th/5th metacarpals.

High PTH and phosphate, low calcium.

How well did you know this?
1
Not at all
2
3
4
5
Perfectly
38
Q

A two-day-old neonate presents with copious purulent eye and nose discharge. Laboratory
examination is performed to diagnose the organism involved.
Which one of the following types of organism would carry the worst prognosis if not treated
immediately?
A. Gram-negative coccobacillus.
B. Gram-negative diplococcus.
C. Gram-positive bacillus.
D. Gram-positive coccus.
E. Intra-cytoplasmic inclusion bodies.

A

B - GN diplococcus

Notes

Neisseria = GN diplococci

Others

GN coccobacillus - enterobacter (e.g. E coli), Hib, pertussis, pseudomonas, legionella, klebsiella
GPB - Diptheria, listeria, clostridia, mycobacteria
GPC - staph, strep
Inclusion bodies - viral

How well did you know this?
1
Not at all
2
3
4
5
Perfectly
39
Q

A three-year-old boy is referred with a history of snoring loudly over the previous nine months. On
history no obstructive apnoea has been noted by his mother. A lateral X-ray of neck shows only
moderate adenoidal hypertrophy with a clearly patent airway. A polysomnogram is reported as
showing an obstructive respiratory disturbance index of 5 events/hr with oxygen desaturation
decreases of 4% of baseline, and an arousal index of 20 events/hr [<10] with reduced rapid eye
movement (REM) sleep.
The most appropriate management of this child is:
A. adeno/tonsillectomy.
B. continuous positive airway pressure therapy.
C. inhaled nasal steroids.
D. reassurance.
E. supplemental oxygen.

A

A - Ts and As

Uptodate

Obstructive sleep apnea (OSA) is characterized by episodes of complete or partial upper airway obstruction during sleep, often resulting in gas exchange abnormalities and arousals, which disrupt sleep. The condition exists in 2 to 5 percent of children and can occur at any age. Untreated pediatric OSA is associated with behavioral and learning problems; in more severe cases, it can be associated with impaired growth (including failure to thrive) and cardiovascular complications. Early diagnosis and treatment may decrease morbidity. Treatment decisions are individualized and depend upon findings from a comprehensive evaluation, including nighttime sleep disruption, daytime dysfunction, physical examination findings, and sleep study findings.

OSA is typically defined by clinically relevant symptoms and an apnea hypopnea index (AHI) >1 or hypoventilation (carbon dioxide >50 mmHg for >25 percent total sleep time) as determined on PSG.

Referral to a specialist for adenotonsillectomy evaluation is generally indicated for otherwise healthy children who have OSA and adenotonsillar hypertrophy (including ≥1+ tonsils). For otherwise healthy children with mild or moderate OSA confirmed by PSG (AHI >1 and <10), watchful waiting with supportive care is a reasonable alternative to adenotonsillectomy. For patients with minimal adenotonsillar tissue or a strong preference for a nonsurgical approach, positive airway pressure therapy is an alternative to adenotonsillectomy.

How well did you know this?
1
Not at all
2
3
4
5
Perfectly
40
Q

A two-month-old boy presents with tachypnoea and poor feeding. He is afebrile. His respiratory rate
is 65/minute with mild intercostal recession, his heart rate is 180/minute and his liver is palpable 4 cm
below the right costal margin. No other abnormalities are identified on clinical examination. His
electrocardiogram (ECG) is shown below.

Which one of the following is the most likely diagnosis?
A. Anomalous left coronary artery.
B. Dilated cardiomyopathy.
C. Hypertrophic cardiomyopathy.
D. Myocarditis.
E. Supraventricular tachycardia.
A

E - SVT

Others
ALCAPA - STE + inverted T waves aVL, V5, V6, presents 6-8 weeks when PVR drops, unsettled with feeds FTT and cardiomegaly.
Dilated cardiomyopathy - multiple transfusions, recent virus, family hx, ECG with evidence of ischaemia, unrecognised tachycardia (exclude ALCAPA on TTE)
Hypertrophic cardiomyopathy - history of sudden unexplained death or cardiomyopathy or myopathy, IDM, more suggestive of metabolic cause, ECG with short PR and giant complexes, QRS-T axis dissociation
Myocarditis - Long PR, low amplitude QRS / T waves

How well did you know this?
1
Not at all
2
3
4
5
Perfectly
41
Q

In which one of the following clinical settings is there the best evidence that prophylactic antibiotics
protect against infection?
A. Cerebrospinal fluid shunt insertion.
B. Chest drain insertion for pneumothorax.
C. Occipital skull fracture.
D. Umbilical artery catheterisation.
E. Urinary catheterisation.

A

A - cerebrospinal fluid shunt insertion

How well did you know this?
1
Not at all
2
3
4
5
Perfectly
42
Q

A 14-year-old girl presents to the emergency department with a 12-hour history of a blistering rash.
The rash is extremely itchy and is spreading up her arm. She is otherwise well and afebrile. A history
is obtained that she had been gardening on the day prior to the rash developing. A diagnosis of an
allergic contact dermatitis is made.

Which one of the following is the most appropriate initial therapy?
A. Intravenous antihistamines.
B. Oral antihistamines.
C. Oral corticosteroids.
D. Subcutaneous adrenaline.
E. Topical corticosteroids.
A

C - oral corticosteroids

Uptodate

Allergic contact dermatitis (ACD) is an acquired, inflammatory reaction of the skin that requires absorption of antigen from the skin surface and recruitment of previously sensitized, antigen-specific T lymphocytes into the skin (delayed type hypersensitivity).

Allergic contact dermatitis (ACD) is commonly encountered by the practicing clinician. The most common clinical expression is an eczematous dermatitis that can be mild to severe, acute and short lived, or chronic. The management of ACD is based upon the identification of the offending allergen, avoidance of exposure, use of safe alternatives, and treatment of symptoms.

Oral corticosteroids are the first line treatment for ACD involving >20 percent of the body surface area or for acute ACD involving the face, hands, feet or genitalia if quick relief is desired (eg, involvement of the eyelids).

Others

  • Topical corticosteroids are the first line treatment for localized ACD
  • Adrenaline for anaphylaxis
  • Because it is T lymphocyte driven, antihistamines have no role
How well did you know this?
1
Not at all
2
3
4
5
Perfectly
43
Q

A 12-year-old boy with cystic fibrosis presents with a six-week history of increased cough and some
wheeze. He has received a two-week course of oral amoxycillin-clavulanic acid, followed by two
weeks of oral ciprofloxacin without any improvement.
His chest X-ray six months ago is shown below (A) with his recent X-ray (B).

[Acute right perihilar opactiy]

Which one of the following is the most likely cause of his deterioration?
A. Allergic broncho-pulmonary aspergillosis.
B. Atelectasis due to mucus plugging.
C. Burkholderia cepacia infection.
D. Staphylococcus aureus infection.
E. Tuberculosis.

A

A - ABPA

MRCPCH

Episodic wheezing, low grade fever, brown sputum, eosinophilia, transient pulmonary infiltrates. Usually in children with CF or asthma. Treat with steroids.

Uptodate

Allergic bronchopulmonary aspergillosis (ABPA) is a complex hypersensitivity reaction in response to colonization of the airways with Aspergillus fumigatus that occurs almost exclusively in patients with asthma or cystic fibrosis (CF). In chronic cases, repeated episodes of bronchial obstruction, inflammation, and mucoid impaction can lead to bronchiectasis, fibrosis, and respiratory compromise.

Although invasive fungal disease is rare in patients with CF, allergic bronchopulmonary aspergillosis (ABPA) is increasingly recognized in CF patients. It can be difficult to distinguish between ABPA and the progressive pulmonary disease that is typical in CF because the symptoms and radiographic features are often similar.

In most CF centers, patients are screened with annual evaluation of total serum IgE; a sudden increase should prompt further investigation for possible ABPA. Patients should also be evaluated for ABPA if they have a marked exacerbation of wheezing or otherwise unexplained deterioration in lung function despite antibiotic therapy.

How well did you know this?
1
Not at all
2
3
4
5
Perfectly
44
Q

A three-month-old boy presents with tachypnoea and failure to thrive. On examination he is noted to
have a respiratory rate of 60/minute with intercostal and subcostal recession. The liver is palpable
4 cm below the right costal margin. The chest is clear and no murmurs are audible. The oxygen
saturation is 89% in room air but increases to 95% with 1 L of subnasal oxygen. His chest X-ray is
shown below.

Which one of the following is the most likely diagnosis?
A. Bronchiolitis.
B. d-transposition of the great arteries and ventricular septal defect.
C. Primary pulmonary lymphangiectasia.
D. Total anomalous pulmonary venous return.
E. Truncus arteriosus.

A

D - TAPVR

MRCPCH

Pulmonary veins drain to either SVC/innominate vein (supracardiac) or liver/IVC (infracardiac). Uncommon. NOT duct dependent.
If obstructed -> presents day 1-7 with cyanosis and collapse.
NO MURMUR.
Signs of right heart failure, breathless, acidotic.
May however present up to 6 months if unobstructed, with murmur or heart failure.
ECG normal in neonate. CXR normal or snowman sign.

Bronchiolitis - infective features e.g. fever, coryza, very unlikely to have a clear chest.
D-TGA with VSD - murmur
C -
E - cyanotic at birth, heart failure in weeks, bounding pulses, harsh murmur

How well did you know this?
1
Not at all
2
3
4
5
Perfectly
45
Q

A three-year-old boy is evaluated because of longstanding hypotonia, delayed motor milestones and
elevated creatine kinase (CK). Family history revealed that during an anaesthetic, his father
developed fever and muscle rigidity. A muscle biopsy was performed and is shown below. The
section is stained with NADH-trichrome.

Which one of the following is the most likely diagnosis?
A. Central core myopathy.
B. Duchenne muscular dystrophy.
C. Myotonia congenita.
D. Nemaline myopathy.
E. Spinal muscular atrophy.
A

A - central core myopathy

Uptodate.

Congenital myopathies are a heterogenous group of hereditary primary muscle disorders that are present from birth, although their onset may be delayed until later in infancy or early childhood. The most common of these rare disorders are nemaline myopathy, central core disease, centronuclear (myotubular) myopathies, and congenital fiber type disproportion.

Congenital myopathies share some common features, though severity is highly variable. Affected individuals usually present at birth or in infancy with hypotonia, weakness, hypoactive deep tendon reflexes, delayed motor milestones, and normal intelligence.

These conditions are caused by genetic abnormalities of muscle development.

Features: neonates, hypotonia+muscle weakness, reflexes present but reduced, predisposition to malignant hyperthermia, usually CK is normal.
These days diagnosis is with genetic testing/sequencing. Muscle biopsy stained with NADH is sometimes valuable.

Others:

  • DMD
  • Myotonia congenita: Defect in chromosome 7, affects chloride channels in muscles, hypertrophy with no weakness, improves with exercise
  • Nemaline: The characteristic rod bodies are best seen with modified Gomori trichrome staining [13]. With this technique, they appear red against a blue-green myofibrillar background.
  • SMA: 5q11, regression, proximal myopathy, fasciculations, no reflexes, hypotonia with facial sparing, CK normal
How well did you know this?
1
Not at all
2
3
4
5
Perfectly
46
Q
Tacrolimus administration after renal transplantation is least likely to cause which of the following?
A. Diabetes mellitus.
B. Hypercholesterolaemia.
C. Hypomagnesaemia.
D. Seizures.
E. Tremors.
A

B - hypercholesterolaemia

Common side effects are hyperglycaemia, renal impairment/AKI, and neurological e.g. tremors, seizures. Diabetes post transplant is common, multifactorial, and incompletely understood.

Other A/E

  • hyperkalaemia (hyporenin/hypoaldosterone type 4 RTA)
  • hypersensitivity
  • HTN
  • infection
  • malignancy
  • hyperkalemia, hypomagnesemia, hyperchloremic metabolic acidosis, hyperuricemia (due to tubular functional alterations)
How well did you know this?
1
Not at all
2
3
4
5
Perfectly
47
Q

A seven-year-old girl is referred to the outpatients department for the evaluation of wheezing episodes
which have occurred every month during the previous three months. She was born at 24 weeks
gestation, ventilated for one month and was on supplemental oxygen for a further 12 weeks. Her
clinical examination is normal. Her baseline lung function tests showed the following:

forced vital capacity (FVC) 80% predicted
forced expiratory volume in one second (FEV1) 70% predicted
maximum mid-expiratory flow (MMEF) 30% predicted

She was treated with prednisolone for two weeks (1 mg/kg/day). Repeat lung function tests post
Ventolin were:

FVC 83% predicted
FEV1 75% predicted
MMEF 40% predicted

Her histamine challenge demonstrated a fall in FEV1 of 25%.

Which one of the following is the most likely diagnosis?
A. Asthma.
B. Bronchial stenosis.
C. Bronchomalacia.
D. Bronchopulmonary dysplasia.
E. Reflux aspiration.
A

D - BPD

FVC 80-120% = normal.
FEV1/FVC = 87.5% which is normal.
No reversibility with bronchodilator (needs to be 12%+).
Positive histamine challenge (>15-20%).

Essentially normal lung function aside from positive histamine challenge. ?Histamine challenge not super valuable:

“The percentage of asthmatics with bronchial hyperresponsiveness, i.e. sensitivity to the test, increased towards 100% on inhaling increasing concentrations of histamine, but this was accompanied by a decrease in specificity and predictive values of positive test in regard to the diagnosis of asthma. However, lower concentrations of histamine may be preferable in order to distinguish between asthma and non-asthma in population samples, as inhalation of 2.4 mg.ml-1 and provocative concentration producing a 20% fall in forced expiratory volume in one second (FEV1) (PC20) provided an acceptable sensitivity (57%), specificity (98%), and predictive value of a positive test (60%). We conclude that as regards the diagnosis of asthma, a low predictive value confirms that the bronchial challenge test plays only a supplementary, but valuable, role in detecting the disease in population samples.”

Given lack of obstructive picture, no responsiveness to bronchodilator, and history of CLD/extreme prem, BPD is most likely.

How well did you know this?
1
Not at all
2
3
4
5
Perfectly
48
Q

The mother of a seven-year-old boy on maintenance chemotherapy for acute lymphoblastic leukaemia
telephones the oncology department to say that a child in her son’s class at school developed
chickenpox yesterday. Her son has no history of having had chickenpox in the past.
Which one of the following actions would you advise for her son?
A. Admission for treatment with intravenous aciclovir.
B. Immediate administration of varicella zoster immune globulin (ZIG).
C. Immediate immunisation with varicella vaccine.
D. Measurement of serum varicella antibody status and administration of ZIG if he is
seronegative.
E. Observe and treat with intravenous aciclovir if chickenpox develops.

A

B - immediate ZIG.

How well did you know this?
1
Not at all
2
3
4
5
Perfectly
49
Q

A male infant was born at term and was well at birth. At four days of age he was discharged from
hospital fully breastfed. Mild umbilical redness was noted at the discharge examination. He returned
a day later with increasing peri-umbilical redness and was commenced on antibiotics. Over the next
24 hours he developed an erythematous rash starting around the perioral area and spreading rapidly
to the face, trunk and flexural areas followed by marked bullous formation and desquamation as
shown below. He was very irritable and had a mild fever.

Which one of the following is the most likely diagnosis?
A. Bullous impetigo.
B. Bullous varicella.
C. Epidermolysis bullosa.
D. Neonatal pemphigus.
E. Staphylococcal scalded skin syndrome.
A

E - SSSS

Caused by exotoxin-producing staphylococci. Localised infection becomes widespread after 24-48 hours. Characteristic signs are fever, skin tenderness, marked erythema, bullae, peeling of the skin. Treat with systemic antibiotics (flucloxacillin) and watch fluid balance.

A bulla is a fluid-filled sac or lesion that appears when fluid is trapped under a thin layer of your skin. It’s a type of blister.

Others

  • impetigo: superficially spreading skin infection characterised by yellow-brown crust. May be bullous. Peaks late summer, common in children <5. Caused by S. aureus or GAS. Treat with oral antis.
  • epidermolysis bullosa: Epidermolysis bullosa (EB) is a group of inherited diseases that are characterised by blistering lesions on the skin and mucous membranes. These may occur anywhere on the body but most commonly appear at sites of friction and minor trauma such as the feet and hands. In some subtypes, blisters may also occur on internal organs, such as the oesophagus, stomach and respiratory tract, without any apparent friction.
How well did you know this?
1
Not at all
2
3
4
5
Perfectly
50
Q

A foetus is found to have an isolated echogenic focus in the left ventricle on a routine 18-week
ultrasound. Which one of the following is the most likely explanation for this finding?
A. A variant of normal.
B. Down syndrome.
C. Neurofibromatosis.
D. Papillary muscle myxoma.
E. Tuberous sclerosis (rhabdomyoma).

A

A - variant of normal

How well did you know this?
1
Not at all
2
3
4
5
Perfectly
51
Q

A six-year-old boy with chronic granulomatous disease presents with a cough of three weeks duration.
A chest X-ray shows right upper lobe consolidation. A computed tomography (CT) scan of the chest
shows destruction of the right second rib.
Infection with which one of the following is most likely to give this clinical picture?
A. Aspergillus.
B. Atypical mycobacteria.
C. Candida albicans.
D. Nocardia.
E. Staphylococcus aureus.

A

A - aspergillus

Disorder of impaired intracellular killing of pathogens and hyperinflammatory state (e.g. IBD, granulomatous lesions). Impaired hydrogen peroxide generation and oxidative burst caused by genetic defects of phox protein subunits constituting the phagocytic NADPH oxidase.
X-linked form accounts for 2/3.
Infections:
- severe bacterial, abscesses, osteomyelitis (staph and pseudomonas)
- supperative adenitis
- pneumonia with pyogenic bacteria, nocardia, aspergillus, burkholderia
- OM
- non typhoidal salmonellosis
- BCG-osis and atypical myobacterial infections
Ix: nitroblue tetrazolium / dihydrorhodamine
Rx: prophylactic antibiotics and antifungals, steroids for granulomas

As per uptodate:

  • lung infections are most common
  • most common organism is aspergillus (Staph second most common)
How well did you know this?
1
Not at all
2
3
4
5
Perfectly
52
Q

The incidence of Grade 3 and 4 acute graft-versus-host disease is greatest when which one of the
following sources of stem cells is utilised?
A. Bone marrow from matched sibling.
B. Bone marrow from matched unrelated donor.
C. Cord blood from matched unrelated donor.
D. Peripheral blood from related matched adult donor.
E. T cell depleted bone marrow from mismatched sibling.

A

B - bone marrow from a matched unrelated donor

Siblings/related always better. Cord blood more compatible.

How well did you know this?
1
Not at all
2
3
4
5
Perfectly
53
Q

A 10-year-old girl has a history of muco-cutaneous candidiasis since age two years and
hypoparathyroidism since age four years. She now has increasing tiredness but is clinically and
biochemically euthyroid.

Which one of the following would be the most useful next test to confirm a diagnosis of adrenal
insufficiency?

A. Adrenal autoantibodies.
B. C26:C22 long chain fatty acid ratio.
C. Plasma adrenocorticotrophic hormone.
D. Plasma cortisol.
E. Plasma renin activity.
A

C - plasma ACTH

Autoimmune polyendocrinopathy candidiasis ectodermal dystrophy (APECED) is an autosomal recessive disease caused by mutations in the autoimmune regulator (AIRE) gene, characterized by the clinical triad of chronic mucocutaneous candidiasis (CMC), hypoparathyroidism, and adrenal insufficiency.

MRCPCH: Autoimmune polyglandular syndrome type 1 = Addison disease, chronic mucocutaneous candidiasis, hypoparathyroidism.

In general testing for adrenal insufficiency relies on demonstrating an inability to respond when stimulated, i.e. the synacthen test. A dose of synthetic ACTH is given and cortisol measured after 30 and 60 minutes. A normal response would be a cortisol level >450-500nmol/L at 30 minutes.

How well did you know this?
1
Not at all
2
3
4
5
Perfectly
54
Q

An asymptomatic infant undergoing a routine discharge check on day two is noted to have a
continuous murmur. His arterial oxygen saturation is 88% in room air.

Which one of the following is the most likely diagnosis?
A. Coronary artery fistula.
B. Persistent ductus arteriosus.
C. Pulmonary atresia and ventricular septal defect.
D. Ruptured sinus of Valsalva.
E. Transposition of the great arteries.

A

C - PA with VSD

Cyanotic lesions: PA with VSD, TGA

Continuous murmur is from a PDA, however a PDA by itself doesn’t cause hypoxia unless there is pulmonary hypertension creating a right to left shunt, otherwise the shunt is left to right hence normal sats.

TGA - more profoundly cyanotic (sats in 50s) given separation of circuits.

Others

  • coronary artery fistula: usually asymptomatic, continuous murmur, bounding pulses, not cyanotic
  • rupture sinus of Valsalva: sudden onset chest pain, signs of severe heart failure and dyspnoea, continuous murmur, more common in Marfans syndrome
How well did you know this?
1
Not at all
2
3
4
5
Perfectly
55
Q

A four-month-old boy is referred for investigation of severe failure to thrive. His weight has
progressively fallen below the 3rd percentile (weight 3.2 kg). He drinks approximately 600 mL of
cows’ milk-based formula per day. He has only mild regurgitation after feeds and no diarrhoea.
On examination he appears generally well in himself and very alert. He has lost most of his
subcutaneous tissue. His abdomen is non-tender without palpable masses or organomegaly.

The most likely diagnosis is:
A. cows’ milk intolerance.
B. cystic fibrosis.
C. diencephalic syndrome.
D. hyperthyroidism.
E. neuroblastoma.
A

C - diencephalic syndrome.

Uptodate

Diencephalic syndrome is a rare consequence of hypothalamic tumors and is characterized by failure to thrive with severe emaciation but normal linear growth, increased appetite, and hyperactivity.

Diencephalic syndrome, a disorder associated with central nervous system tumors, is a rare cause of profound weight loss related to increased energy expenditure but should not be overlooked.

Poor weight gain by age as per uptodate for birth-6mo:

  • Poor quality of suck (whether breast- or bottle-fed) and/or oral motor dysfunction
  • Improper formula preparation
  • Breastfeeding problems, including insufficient milk supply
  • Inadequate number of feedings
  • Poor feeding interactions (eg, infant gags or vomits during feeding, caretaker misreads signals of hunger or satiety)
  • Child neglect
  • Parental mental illness
  • Metabolic, chromosomal, or anatomic abnormalities
  • Underfeeding (possibly related to poverty)
  • Milk protein intolerance
  • Cystic fibrosis
  • Congenital heart disease
  • Gastroesophageal reflux disease

Others
A - unlikely given cows milk intake with no reflux or diarrhoea
B - surely more likely than diencephalic syndrome
D - neonatal hyperthyroidism/Graves results from maternal Graves, not mentioned here
E - no palpable abdominal mass

How well did you know this?
1
Not at all
2
3
4
5
Perfectly
56
Q

A 10-year-old obese boy presents with high fevers, shortness of breath, and he appears quite unwell.
A chest X-ray reveals bilateral pleural effusions and a large heart. Ultrasound confirms a pericardial
effusion. There is no pulsus paradoxus. He is started on intravenous cefotaxime. He becomes more
short of breath and remains oxygen dependent after 24 hours, with his chest X-ray showing increased
pleural effusions. There are no changes in blood pressure.

The priority in his management is to:
A. add intravenous flucloxacillin.
B. insert large pleural chest drains.
C. observe in intensive care unit.
D. perform a thoracotomy to create a pericardial window.
E. perform pleural and pericardial paracentesis.

A

B - insert large pleural chest drains. Possibly because no tamponade/pulsus paradoxus? Otherwise would need pericardiocentesis.

Uptodate

Pulsus paradoxus: Systemic blood pressure is not constant but varies slightly from heart beat to heart beat and between inspiration and expiration. Normally, the systolic blood pressure decreases by less than 10 mmHg during inspiration, but a decline of this magnitude is not detectable on examination of the peripheral pulse. Moderate to severe cardiac tamponade, and occasionally constrictive pericarditis, induce hemodynamic changes that enhance the inspiratory fall in systolic blood pressure. This exaggerated drop in systemic blood pressure during inspiration is termed pulsus paradoxus. For patients without an indwelling arterial catheter, pulsus paradoxus must be measured by the clinician using a manually-operated sphygmomanometer. To measure pulsus paradoxus, a manually-operated sphygmomanometer is employed for blood pressure measurement in the standard fashion except that the cuff is deflated more slowly than usual. During deflation, the first Korotkoff sounds are audible with heart beats occurring only during expiration. With further cuff deflation, Korotkoff sounds become audible on all heart beats throughout the respiratory cycle. The difference between the systolic pressure at which the first Korotkoff sounds are heard during expiration and the pressure at which they are heard throughout the respiratory cycle quantifies pulsus paradoxus. Severe pulsus paradoxus, variably defined as >10 to 20 mmHg, can be palpated in the radial, brachial, or femoral pulses as a weakening or disappearance of the pulse during inspiration.

The presence of a pulsus paradoxus greater than 10 mmHg increases the likelihood of tamponade by 3.3-fold, while its absence greatly lowers but does not completely eliminate the possibility of cardiac tamponade.

How well did you know this?
1
Not at all
2
3
4
5
Perfectly
57
Q

The parents of a 15-year-old boy telephone about his changing behaviour over the previous two
weeks. His teachers have described poor concentration and disruptive behaviour at school and he
has been truant for most of the past week. He has been very difficult to live with, arguing with his
older brother, watching television all night and going out during the day. The parents of his girlfriend
are also concerned as they believe the boy and their daughter have started having unprotected sex
and he has encouraged her to leave school. The boy refuses to see you.

Which one of the following diagnoses best explains these symptoms?
A. Attention deficit/hyperactivity disorder.
B. Bipolar affective disorder.
C. Conduct disorder.
D. Marijuana abuse.
E. Schizophreniform disorder.

A

B - bipolar affective disorder. Not really sure why, can’t see anything in the stem about mania or depression. Might be outdated? 2002.

How well did you know this?
1
Not at all
2
3
4
5
Perfectly
58
Q

Which one of the following is the least reliable predictor of neurodevelopmental outcome in term
infants with hypoxic-ischaemic encephalopathy?
A. Age at onset of seizures of less than 12 hours.
B. An abnormal electroencephalogram (EEG) at seven days of age.
C. Anuria or oliguria for more than 24 hours.
D. Apgar score at 10 minutes.
E. Neurological examination at 10 days of age.

A

D - APGAR score at 10 minutes

How well did you know this?
1
Not at all
2
3
4
5
Perfectly
59
Q

A 15-year-old boy presents to the clinic with a three-month history of intermittent left knee pain at night
subsequent to being struck on the knee by a cricket ball which resulted in extensive bruising. One
week prior to presentation he noticed a swelling just above the knee. An X-ray of his left knee is
shown below.

Which one of the following is the most likely diagnosis?
A. Aneurysmal bone cyst.
B. Chronic osteomyelitis.
C. Ewing sarcoma.
D. Osteochondroma.
E. Osteogenic sarcoma.
A

E - osteogenic sarcoma (AKA osteosarcoma).

How well did you know this?
1
Not at all
2
3
4
5
Perfectly
60
Q
Stevens-Johnson syndrome is most likely to occur with which one of the following anticonvulsant
drugs?
A. Carbamazepine.
B. Lamotrigine.
C. Phenytoin.
D. Sodium valproate.
E. Vigabatrin.
A

B - lamotrigine

Uptodate

Stevens-Johnson syndrome/toxic epidermal necrolysis — Stevens-Johnson syndrome/toxic epidermal necrolysis (SJS/TEN) is a severe mucocutaneous eruption that is frequently triggered by medications. Allopurinol, certain antiepileptics, antibacterial sulfonamides, and oxicam nonsteroidal anti-inflammatory drugs (NSAIDs) are most frequently implicated. This disorder is characterized by epidermal necrosis and sloughing of the mucous membranes and skin. The amount of skin detachment related to the body surface area is used to distinguish SJS from TEN; detachment affects less than 10 percent of the body surface in SJS and more than 30 percent in TEN.

How well did you know this?
1
Not at all
2
3
4
5
Perfectly
61
Q

An 18-month-old girl develops fever and diarrhoea. Two days later she begins to vomit, develops a
faint, generalised maculopapular rash, increasing tachypnoea and becomes obtunded. She is febrile
39.2°C, pale and poorly perfused. Her capillary refill time is 4 seconds. Her pulse is 140/minute and
thready. Blood pressure is 90/60 mmHg. Her apex beat is difficult to feel but is in the mid-axillary line.
She has a pansystolic murmur radiating to the axilla. Her respiratory rate is 52/minute and chest
clear. Her liver is palpable 5 cm below the costal margin. She is drowsy and has moderate neck
stiffness. Her generalised rash is truncal and is fine, maculopapular and blanches on pressure.
Investigations show:
haemoglobin 108 g/L [110-140]
white cell count 11.4 x 109/L [4.0-11.0]
neutrophils 5.2 x 109/L [1.0-4.0]
platelet count 201 x 109/L [150-400]
liver function tests normal

cerebrospinal fluid (CSF)
white cell count 420 x 106/L (300 polymorphs, 120 monocytes)
red cell count 20 x 106/L
sugar 2.2 mmol/L (blood sugar 4.2 mmol/L)
protein 0.5 mmol/L [0.2-0.6]
CSF Gram stain no organisms seen

chest X-ray cardiomegaly with pulmonary congestion

Which one of the following is the most likely causative organism?
A. Coxsackie B virus.
B. Enterovirus 71.
C. Herpes simplex virus.
D. Mycoplasma pneumoniae.
E. Neisseria meningitidis.
A

A - coxsackie B virus. Given CSF results (Gram stain, protein, sugar) in keeping with viral, + heart/lung involvement, group B enteroviruses/coxsackievirus are most common cause. Enterovirus 71 causes HFM, and if involving CNS preferentially targets motor nuclei/anterior horn cells causing paresis.

Uptodate

Spectrum of disease — More than 90 percent of infections caused by the non-polio enteroviruses are asymptomatic or result only in an undifferentiated febrile illness. When more serious disease occurs, the clinical spectrum and disease severity vary with the age, gender, and immune status of the host. Some clinical syndromes (viral meningitis and some exanthema) are caused by numerous enterovirus serotypes, while others appear limited to specific enterovirus subgroups (eg, hand, foot, and mouth disease [HFM] with enterovirus A71 and some group A coxsackieviruses, and pleurodynia and myocarditis with the group B coxsackieviruses).

Central nervous system infections — Acute enterovirus infection of the CNS occurs at all ages. Meningitis is the most common CNS manifestation. Both generalized and focal encephalitis occur less frequently. Certain enteroviruses (ie, polioviruses, enterovirus D68, enterovirus A71) preferentially target the motor nuclei and anterior horn cells of the brainstem and spinal cord, causing acute paresis of cranial and spinal nerves.

Viral (aseptic) meningitis — Aseptic meningitis affects persons of all ages but is most commonly observed in infants less than one year of age. The enteroviruses cause more than 90 percent of cases in infants; the majority are due to species B enteroviruses, which include the group B coxsackieviruses and most echoviruses.

RCH - CSF interpretation
Bacterial: WCC >100 and predominantly neutrophils, usually <100 lymphocytes, protein >1, glucose CSF:blood <0.4
Viral: WCC elevated, predominantly lymphocytes, protein 0.4-1, glucose normal

How well did you know this?
1
Not at all
2
3
4
5
Perfectly
62
Q
Abuse of which one of the following recreational drugs during pregnancy presents the greatest risk of
birth defects to the foetus?
A. Amphetamines.
B. Cocaine.
C. Heroin.
D. THC (tetrahydrocannibol).
E. Tobacco.
A

B - cocaine. ?Outdated. Cocaine crosses the placenta and fetal blood-brain barrier; vasoconstriction is the major purported mechanism for fetal and placental damage. Teratogenic effects have not been definitively proven.
Doesn’t seem to be definitive evidence that any of the listed drugs increase the risk of birth defects.

Uptodate

THC/cannabis - Available data do not suggest an increase in congenital anomalies among children born to marijuana users.
Amphetamines - Amphetamines and their byproducts cross the placenta. No fetal structural abnormalities have been definitively associated with perinatal amphetamine exposure.
Opioids - Multiple obstetric complications have been associated with opioid use disorder in pregnancy. Nothing listed about congenital anomalies of the foetus.
Smoking - Although the overall rate of congenital malformations does not appear to be higher among infants born to pregnant individuals who smoke, smoking may increase the risk of specific anomalies. Studies have reported links between maternal smoking and the development of cleft lip with or without cleft palate, gastroschisis, anal atresia, transverse limb reduction defects, cardiac defects, digital anomalies (polydactyly, syndactyly, or adactyly), and bilateral renal agenesis or hypoplasia.

How well did you know this?
1
Not at all
2
3
4
5
Perfectly
63
Q

A five-year-old girl of Indian origin presents with three months of increasing lethargy and fatigue, with
occasional fevers. Previously active, she is increasingly reluctant to get up in the morning or walk,
and cries and complains that she hurts all over. A lump has appeared in her armpit and discharges
white material intermittently (see photograph shown above). The family moved from India two years
ago. She is fully immunised. There is no history of tuberculosis (TB) in the family or of TB contact.
On examination she has difficulty rising from the floor unaided and has generalised weakness most
marked in the hips and shoulders. The lesion in the axilla is firm and non-tender with induration of the
underlying skin.

Which one of the following is the most likely diagnosis?
A. Dermatomyositis.
B. Relapsing panniculitis (Weber-Christian disease).
C. Systemic lupus erythematosus.
D. Tuberculosis.
E. Visceral larva migrans.

A

A - dermatomyositis. Other options don’t really explain widespread weakness.

MRCPCH

Non-suppurative myositis with characteristic skin rash and vasculitis. Occurs in girls more commonly. Peak incidence 4-10 years.

Features:

  • muscle pain and occasional tenderness
  • muscle weakness (limb, girdle, neck, palate, swallowing)
  • oedema
  • rash (periorbital heliotrope eruption and oedema)
  • deep red patches over extensor surface of finger joints (Gottron patches), elbows, kness, ankles
  • vasculitis, skin ulceration
  • nailfold and eyelid (dilated capillaries)
  • retinitis in some
  • myocarditis with arrhythmias can occur
  • arthralgia/arthritis with contractures
  • limited joint mobility
  • GI dysfunction
  • pulmonary involvement
  • calcinosis after 1-2 years

Investigations:

  • MRI shows inflammation
  • ESR usually normal
  • serum muscle enzymes elevated
  • EMG shows denervation/myopathy
  • biopsy shows inflammation or fibre necrosis and small vessel occlusive vasculitis
  • ANAs positive in some

Prognosis
- variable, usually good with adequate treatment

Management

  • PT, splinting
  • steroids
  • cytotoxic drugs if required
  • anti-TNF
  • careful monitoring essential

Others:

  • Panniculitis refers to a group of conditions characterised by inflammation of the fat layer below the skin (sub-cutaneous fat).
  • Visceral larva migrans (VLM) is a condition in humans caused by the migratory larvae of certain nematodes, humans being a dead-end host, and was first reported in 1952. Nematodes causing such zoonotic infections are Baylisascaris procyonis, Toxocara canis, Toxocara cati, and Ascaris suum.
How well did you know this?
1
Not at all
2
3
4
5
Perfectly
64
Q

Which one of the following is least likely to be associated with Munchausen by proxy syndrome?
A. Father frequently absent.
B. Maternal intravenous drug abuse.
C. Older sibling died from sudden infant death syndrome aged six months.
D. Recurrent polymicrobial bacteraemia.
E. Recurrent unwitnessed seizures.

A

B - maternal IVDU

Uptodate:

Perpetrator features — Limited evidence suggests some common features among perpetrators. Systematic study of mothers who medically abused their children have focused on perpetrators of severe abuse and found the following associations:
●Over 95 percent are female, mostly mothers or primary caretakers, although cases of collusion with other caregivers, including male members of the household have been described
●History of factitious or somatoform disorders
●History of unfortunate childhoods (eg, deprivation and abuse)
●Past history of self-harm, alcohol or drug abuse, and criminal activity

How well did you know this?
1
Not at all
2
3
4
5
Perfectly
65
Q

The emergence of variant Creutzfeldt-Jakob disease (vCJD) in the United Kingdom has resulted in
changes to the Australian Red Cross and New Zealand Blood Services’ procedures including a
deferral of blood donations from people who have been in the United Kingdom for six months or more
between 1980 and 1996. The main reason for this deferral is:

A. blood from a sheep incubating bovine spongiform encephalopathy (BSE) was able to cause
the disease when transfused into another sheep.
B. blood from experimentally infected primates with vCJD has been shown to transmit the
disease.
C. blood from human beings with clinically evident vCJD has been reported to transmit the
disease after intracerebral inoculation into animals.
D. there is evidence that a blood donor from the United Kingdom who was incubating vCJD has
transmitted the disease to a recipient.
E. tonsils from patients with vCJD contain the prion protein which is thought to be a good index of
infectivity.

A

A - blood from a sheep incubating bovine spongiform encephalopathy was able to cause the disease when transfused into another sheep.

How well did you know this?
1
Not at all
2
3
4
5
Perfectly
66
Q
Which one of the following sleep behaviours has the highest prevalence in primary school aged
children?
A. Bruxism.
B. Enuresis.
C. Night terrors.
D. Sleep talking.
E. Sleep walking.
A

A - bruxism

Uptodate:

Sleep-related bruxism involves activation of the masticatory muscles, resulting in tooth clenching and grinding during sleep. The prevalence of bruxism during sleep peaks in childhood and progressively declines with age. Although often asymptomatic, frequent bruxism may become clinically significant when it interferes with sleep or results in tooth wear or jaw discomfort.

Based largely on cross-sectional survey studies and self-report, sleep-related bruxism affects 15 to 40 percent of children.

RCH:

At 4 years of age, nearly 1 in 3 children wets the bed, but this falls to about 1 in 10 by age 6.

How well did you know this?
1
Not at all
2
3
4
5
Perfectly
67
Q

A 14-year-old girl presents with bitemporal hemianopia. Her magnetic resonance imaging (MRI) scan
is shown above. She has grown poorly for five years and has a history of polydipsia and polyuria. Her
height is <1st percentile, weight on the 3rd percentile and she has no signs of puberty.
Investigations reveal:

computed tomography (CT) scan of head mass as seen on MRI, no calcification
free thyroxine (free T4) 8 pmol/L [10-20]
thyroid-stimulating hormone (TSH) 2.50 mU/L [0.03-4.50]
urea and electrolytes normal
urine specific gravity 1.02 [1.05-1.25]
beta human chorionic gonadotrophin detected

Which one of the following is the most likely diagnosis?
A. Craniopharyngioma.
B. Germinoma.
C. Glioma.
D. Haemangioma.
E. Hamartoma.
A

B - germinoma. Even without the image bhCG is a bit of a giveaway.

MRCPCH

Tumours derived from germ cells can be gonadal (30%) or extragonadal (70%). Extragonadal sites are the sacrococcygeal region, retroperitoneum, mediastinum, neck, and pineal area of the brain.

Gonadal tissue can give rise to any cell type, so tumours express any cell line in any stage of differentiation.

Presenting features vary with site of original. Occasionally endocrinology-related pathology with abnormal or precocious puberty.

Serum markers alpha-fetoprotein and beta-human chorionic gonadotrophin are useful in diagnosing and monitoring disease state.

How well did you know this?
1
Not at all
2
3
4
5
Perfectly
68
Q

You are asked to see a well neonate with a birthmark (shown above - port wine stain to face).
Which one of the following would be the most important next step in evaluation of this child?
A. Cardiac assessment.
B. Chromosomal analysis.
C. Magnetic resonance imaging (MRI) scan of the brain.
D. Ophthalmological assessment.
E. Renal ultrasound.

A

D - ophthalmological assessment. ?SWS -> glaucoma.

MRCPCH

Sturge Weber syndrome

  • glaucoma 50% before the age of 2
  • choroidal haemangiomas 40%

Uptodate:

The predominant ocular abnormality of SWS is glaucoma (increased intraocular pressure), which occurs in 30 to 70 percent of affected patients. Therefore, we suggest ophthalmology referral and follow-up for all patients presenting with SWS and for all infants and children presenting with a capillary malformation involving the upper and lower eyelids and/or extensive lesions in the frontotemporal region (V1 and V2 segments).

Sturge-Weber syndrome (SWS) is a rare congenital vascular disorder characterized by facial capillary malformation (port wine stain) and associated capillary-venous malformations affecting the brain and eye. It is not a heritable disorder. Thus, recurrence is unlikely.

SWS is characterized by a facial capillary malformation, also known as a port wine stain, and an associated leptomeningeal capillary-venous malformation (leptomeningeal angioma) involving the brain and eye. These vascular malformations are associated with specific neurologic and ocular abnormalities.

The neurologic features of SWS may be progressive and include seizures, focal neurologic deficits, and intellectual disability. Visual field defects are common when the occipital cortex is affected. These manifestations occur with variable severity. Hydrocephalus also may occur. This complication is thought to result from increased venous pressure caused by thrombosis of the deep venous channels or extensive arteriovenous anastomoses. A small proportion of patients have no neurologic abnormalities.

Ocular features of SWS include glaucoma and capillary-venous vascular malformations of the conjunctiva, episclera, choroid, and retina.

How well did you know this?
1
Not at all
2
3
4
5
Perfectly
69
Q

A 14-year-old girl with cystic fibrosis is reviewed in clinic. It is two months since her last review and in
that time she has lost 2 kg in weight, and has had a 10% reduction in forced expiratory volume in one
second (FEV1) but has had no recent wet cough. She has had normal stools. On examination her
chest is clear.

Which one of the following is the most likely diagnosis?
A. Diabetes mellitus.
B. Gastro-oesophageal reflux.
C. Inadequate pancreatic supplementation.
D. Inadequate salt intake.
E. Recurrent active bronchitis.

A

A - diabetes mellitus. Others less likely as essentially asymptomatic.

Uptodate

Cystic fibrosis-related diabetes mellitus — CFRD is a common cause of nutritional decline in individuals with CF and is also associated with declines in pulmonary function; treatment with insulin attenuates or reverses these effects. The prevalence of CFRD rises markedly with age, so that CFRD affects approximately 15 percent of adolescents with CF and almost 50 percent of adults over age 30 years. Patients with CF should be screened for CFRD using periodic oral glucose tolerance testing starting at 10 years of age.

Others

  • Patients with pancreatic insufficiency characteristically have frequent, bulky, foul-smelling stools that may be oily. Older children may also report that their stools float or stick to the toilet bowl (reflecting their high fat content).
  • Acute exacerbations with cough, tachypnea, dyspnea, increased sputum production, malaise, anorexia, and weight loss
  • Occasionally, individuals with CF may develop subacute or chronic hypovolemia with hyponatremia, hypochloremia, hypokalemia, and metabolic alkalosis (sometimes known as pseudo-Bartter syndrome). In contrast with Bartter syndrome, urinary chloride excretion is low. This condition is caused by excessive loss of sodium and chloride in sweat and may develop in CF patients with inadequate sodium intake.
How well did you know this?
1
Not at all
2
3
4
5
Perfectly
70
Q

Anorexia nervosa has a highly variable outcome. Which one of the following factors is least predictive
of a poor prognosis?
A. Degree of weight loss at initial presentation.
B. Frequent hospitalisations.
C. Highly disturbed relationships with family members.
D. Prepubertal onset.
E. Poor premorbid functioning.

A

A - degree of weight loss at presentation

MRCPCH

Good prognosticators

  • younger age at onset (?but not prepubertal…)
  • less denial
  • improved self-esteem

Poor prognosticators

  • parental conflict
  • bulimia
  • coexisting behavioural disorders
How well did you know this?
1
Not at all
2
3
4
5
Perfectly
71
Q

A male infant is born at term following an uncomplicated pregnancy. He has Apgar scores of 8 at 1
and 5 minutes. Severe hypotonia is present from birth. Feeding is difficult to establish and tube
feeding is required. However, no respiratory support is needed. On examination at six weeks of age,
he remains very hypotonic and is still being tube fed. His reflexes are normal and despite his
hypotonia, he is observed to move all limbs and has a normal cry.
The investigation which is most likely to make a diagnosis in this infant is:
A. acetylcholine receptor studies.
B. deletion studies for spinal muscular atrophy.
C. dystrophin deletion studies.
D. fluorescent in-situ hybridisation (FISH) for deletion on chromosome 15p.
E. mutation testing for myotonic dystrophy.

A

D - FISH for deletion on chromosome 15p

?Prader Willi Syndrome

MRCPCH

Hypotonia in neonates:

  • Central causes: encephalopathy, IVH, infection, T21/13/18, structural brain abnormalities, metabolic disease, drugs, PWS, hypothyroidism, kernicterus (early)
  • Spinal cord lesions: trauma, tumours, cysts, vascular malformations
  • NMD: SMA, congenital myotonic dystrophy, congenital myopathies, myasthenia gravis

PWS

  • clinical: neonatal hypotonia and poor feeding
  • moderate learning disability
  • hyperphagia and obesity later in childhood
  • small genitalia
  • 70% deletion on paternal chromosome 15
How well did you know this?
1
Not at all
2
3
4
5
Perfectly
72
Q

A 13-year-old girl presents with a four-week history of progressive deterioration with lethargy,
occasional vomiting and peripheral oedema. Urinalysis shows 4+ blood and 4+ protein.
The following blood results are obtained:
haemoglobin 78 g/L [115-150]
plasma potassium 7.0 mmol/L [3.5-5.0]
plasma creatinine 0.60 mmol/L [0.04-0.08]
plasma urea 60.0 mmol/L [3.2-7.7]
Her renal biopsy (shown below) shows crescentic glomerulonephritis.

Her clinical progress over the next three months is most likely to show which one of the following?
A. Complete and spontaneous recovery of renal function.
B. Dialysis dependence.
C. Good response to high dose cyclosporin A.
D. Persistent heavy proteinuria.
E. Recurrent macroscopic haematuria.

A

D - persistent heavy proteinuria

Rapidly progressive GN — Rapidly progressive glomerulonephritis (RPGN) is a clinical syndrome manifested by features of subacute nephritic syndrome with a progressive loss of kidney function over a comparatively short period of time (days, weeks, or months). It is characterized morphologically by extensive crescent formation.

RPGN occurs rarely in children. Causes of pediatric RPGN include:
●Primary GN – IgA nephropathy, MPGN, and anti-glomerular basement membrane (GBM) disease
●Secondary GN – Antineutrophil cytoplasmic autoantibody (ANCA)-associated vasculitis, lupus nephritis, poststreptococcal GN, IgAV (HSP) nephritis

Early diagnosis with kidney biopsy and serologic testing, and early initiation of appropriate therapy are essential to minimize the degree of irreversible kidney injury. Empiric therapy may be started in patients with severe disease, particularly if either kidney biopsy or interpretation of the biopsy will be delayed. However, despite aggressive treatment, approximately half of the affected children will develop end-stage kidney disease (ESKD).

How well did you know this?
1
Not at all
2
3
4
5
Perfectly
73
Q

Which one of the following is the most common among adolescent (13-17 years) girls in the
community?
A. Anorexia nervosa.
B. Anxiety disorders.
C. Attention deficit/hyperactivity disorder.
D. Conduct disorder.
E. Depressive disorders.

A

B - anxiety disorders

Uptodate: Anxiety disorders are the most common psychiatric disorders with onset in childhood, with prevalence estimates ranging from 10 to 30 percent.

The reported prevalence of ADHD in children varies from 2 to 18 percent depending upon the diagnostic criteria and the population studied (eg, primary care versus referral). The prevalence in school-age children is estimated to be between 9 and 15 percent, making it one of the most common disorders of childhood.

MRCPCH

Depression - 2-3/100 teenagers
Anorexia - 1%
Conduct disorder - 4%, but strong male predominance

74
Q

Which one of the following drug associations is least likely to occur?
A. Chloramphenicol and grey baby syndrome.
B. Fusidic acid and jaundice.
C. Rifabutin and red/orange skin discolouration.
D. Rifampicin and orange tears.
E. Teicoplanin and red man syndrome.

A

E - teicoplanin and red man syndrome

RMS is due to vancomycin - direct activation of mast cells.

75
Q

A 12-year-old boy has a renal ultrasound performed as part of an evaluation for an acute abdomen.
He is otherwise well and there is no past medical history of note.
Examination shows a well-grown boy with no significant physical abnormalities. His ultrasound is
shown below.

Which one of the following is he least likely to develop?
A. Colonic diverticula.
B. Hepatic cysts.
C. Periportal fibrosis.
D. Polyuria.
E. Renal failure.
A

C - periportal fibrosis - a manifestation/association of cystic fibrosis.

ADPKD

Uptodate

Nonrenal manifestations — Clinical manifestations that involve nonrenal organs include:
●Cardiac abnormalities – A study from the previously mentioned prospective cohort reported increased left ventricular mass index in children with blood pressure above the 75th percentile for age, sex, and height prior to any observed increase in kidney volume. In addition, another study from the same group reported a 12 percent incidence of mitral valve prolapse in children with ADPKD compared with only 3 percent of unaffected children.
●Cysts in other organs – Magnetic resonance (MR) studies have shown the presence of hepatic cysts in 55 percent of patients before the age of 25 years. However, pancreatic, ovarian, intestinal, and splenic cysts are rarely observed in children.
●Cerebral aneurysms – Aneurysmal rupture in childhood resulting in a subarachnoid or intracerebral hemorrhage is extremely rare and screening is not recommended in children.

76
Q
For which one of the following conditions is there the best evidence of benefit from systemic
corticosteriods?
A. Acute disseminated encephalomyelitis.
B. Cerebral malaria.
C. Guillain-Barré syndrome.
D. Meningococcal meningitis.
E. Tuberculous meningitis.
A

E - TB meningitis

77
Q
Which of the following techniques is most appropriate for testing visual acuity in a four-year-old child?
A. Cover test.
B. E test.
C. Red reflex test.
D. Snellen chart.
E. Toy matching test.
A

B - E test. Google: “The highest test that the child is capable of performing should be used; in general, the tumbling E chart should be used for children aged 3 to 5 years and Snellen letters or numbers for children aged 6 years and older.”

Wikipedia

An E chart, also known as a tumbling E chart, is an ophthalmological chart used to measure a patient’s visual acuity.

This chart is useful for patients who are unable to read the Latin alphabet – for example, very young children. It is also used in countries where people do not use the alphabet in their native language – for example, in China.

It contains rows of the letter “E” in various kinds of rotation. The patient is asked to state (usually by pointing) where the limbs of the E are pointing, “up, down, left or right.” Depending on how far the patient can “read”, his or her visual acuity is quantified. It works on the same principle as Snellen’s distant vision chart.

Others

  • Cover testing is the gold standard objective method for determining the presence, type, and magnitude of ocular misalignment (strabismus) (cover each eye and see if the uncovered eye moves/realigns)
  • Red reflex test is for neonates
  • Snellen chart too young
  • Toy matching test younger than this ?toddlers
78
Q
Which of the following classes of immunoglobulins has the longest mean half-life?
A. IgA.
B. IgD.
C. IgE.
D. IgG.
E. IgM.
A

D - IgG.

In this way we calculated the half-life of IgM as 2 days, IgG3 and IgG1 as 6-8 days, IgG2b has a half-life of 4-6 days. IgE has a half-life of 12 h. A polymeric form of IgA was found to be eliminated from the serum with a half-life of 17-22 h

79
Q

Allopurinol is most likely to precipitate methotrexate toxicity by which one of the following mechanisms?
A. Displacement from protein binding sites.
B. Impairment of renal excretion.
C. Inhibition of dihydrotetrafolate reductase.
D. Inhibition of metabolism.
E. Promotion of gastrointestinal absorption.

A

B - impairment of renal excretion

Methotrexate is excreted primarily by the renal route. Renal dysfunction can cause delayed methotrexate elimination, which can result in a marked increase in hematological and nonhematological toxicity, and may delay subsequent therapy. We found that patients receiving allopurinol had a significantly lower median methotrexate clearance and were more likely to experience methotrexate-induced mucositis than were those receiving urate oxidase. The decrease in methotrexate clearance in patients treated with allopurinol may be explained by a weaker hyporuricemic effect of allopurinol, leading to precipitation of uric acid in renal tubules.

80
Q
A two-week-old baby, otherwise well, has a warm, erythematous and enlarged breast bud that is discharging pus. The infant is most likely to be infected with:
A. Escherichia coli.
B. Group A Streptococcus.
C. Group B Streptococcus.
D. Staphylococcus aureus.
E. Staphylococcus epidermidis.
A

D - staph aureus

Uptodate

Mastitis and breast abscess are less common in infants ≥2 months than in younger infants. Most cases of mastitis and breast abscess in infants are caused by Staphylococcus aureus. Less common causes include gram-negative enteric organisms (eg, Escherichia coli, Salmonella), anaerobes, and group B Streptococcus (Streptococcus agalactiae). The pathogenesis of infant mastitis typically involves spread of pathogenic bacteria from the skin and/or mucous membranes to the breast parenchyma through the nipple.

In infants younger than two months, the main consideration in the differential diagnosis of mastitis is physiologic breast hypertrophy, which resolves spontaneously. In contrast to mastitis, physiologic breast hypertrophy usually is bilateral and symmetric; the breasts are neither erythematous nor tender; if present, the nipple discharge is milky rather than purulent and does not contain polymorphonuclear white blood cells or bacteria on Gram stain.

81
Q

Which of the following tests is most likely to detect early respiratory failure in neuromuscular disease?
A. Blood gas analysis.
B. Forced expiratory volume in one second (FEV1).
C. Forced vital capacity (FVC).
D. Maximal inspiratory/expiratory airway pressures.
E. Polysomnography.

A

E - polysomnography

82
Q
Which of the following best describes the mechanism of resistance of Haemophilus influenzae to ampicillin?
A. Altered binding proteins.
B. Decreased permeability of cell wall.
C. Efflux pump.
D. Production of beta lactamase.
E. Pus formation.
A

D - production of beta lactamase

83
Q

Which of the following correlates most strongly with language abilities at age six years?
A. Cumulative time with otitis media with effusion since birth.
B. Intelligence quotient.
C. Number of siblings.
D. Socio-economic status.
E. Time spent in child care.

A

B - intelligence quotient

84
Q

A three and a half-year-old boy presents after falling off a coffee table at home. He has sustained an injury
to his right elbow and is complaining of a ‘funny feeling’ in his right thumb and forefinger.
On examination he has obvious deformity of his right elbow with a strong radial pulse and normal capillary
refill of his nail beds. An X-ray of his elbow is shown below.

(Distal humerus fracture)

Which of the following movements is most likely to be affected on clinical examination?
A. Abduction of the fingers.
B. Adduction of the fingers.
C. Thumb abduction.
D. Thumb adduction.
E. Wrist extension.
A

C - thumb abduction

Median nerve

  • flex wrist
  • pronate
  • thumb abduction, opposition, flexion
  • also innervates first two lumbricals (flex MCP joint with extension at IP joints)
85
Q

A one-month-old infant presents with stridor, which is worse when crying. A barium swallow demonstrates
an anterior indentation of the oesophagus, while bronchoscopy demonstrates a pulsation of the posterior
wall of the lower trachea and also tracheomalacia.
Which of the following is the most likely cause of these findings?
A. Aberrant left pulmonary artery.
B. Aberrant right subclavian artery.
C. Anomalous innominate artery.
D. Anomalous left carotid artery.
E. Double aortic arch.

A

A - aberrant left pulmonary artery. Left pulmonary artery courses between trachea and oesophagus.

Double arch - bilateral compression oesophagus, anterior compression trachea
Anomalous innominate artery - right brachiocephalic artery arises distally and tracks back across front of trachea, therefore normal oesophagus but anterior compression of trachea
Aberrant right subclavian artery - right subclavian artery arises distally and swings behind oesophagus, therefore normal trachea indented oesophagus

86
Q
Which of the following infectious agents is most commonly associated with erythema multiforme?
A. Epstein Barr virus.
B. Group A Streptococcus.
C. Herpes simplex virus.
D. Mycoplasma pneumoniae.
E. Parvovirus B19.
A

C - HSV

MRCPCH

Acute self-limiting onset of symmetrical fixed red papules, some of which form target lesions. May blister. Can show Koebner phenomenon. May involve lips, buccal mucosa, tongue.

Causes

  • infections: HSV, Mycoplasma, EBV, chlamydia
  • drugs: sulfonamides, penicillin
  • collage disease: SLE, polyarteritis nodosa
  • malignancy

Uptodate

Erythema multiforme (EM) is an acute, immune-mediated condition characterized by the appearance of distinctive target-like lesions on the skin. These lesions are often accompanied by erosions or bullae involving the oral, genital, and/or ocular mucosae.

A variety of factors have been implicated in the pathogenesis of EM. The disorder is most commonly induced by infection, with herpes simplex virus being the most frequent precipitator. The clinical course of EM is usually self-limited, resolving within weeks without significant sequelae. Infections (viral, bacterial, or fungal) account for approximately 90 percent of cases.

87
Q
Growth hormone is secreted from the anterior pituitary gland. Which of the following is least likely to
stimulate growth hormone release?
A. Exercise.
B. High carbohydrate intake.
C. High protein intake.
D. Hypoglycaemia.
E. Sleep.
A

B - high cardbohydrate intake

88
Q

A mother brings her nine-month-old daughter to see you with concerns about her child’s hearing. Which of
the following would be the most appropriate to determine the child’s hearing acuity?
A. Auditory brainstem response (ABR) testing.
B. Automated auditory brainstem response (A-ABR) testing.
C. Kendall toy test.
D. Tympanometry.
E. Visual reinforcement audiometry.

A

E - visual reinforcement audiometry. 6mo-2yrs.

ABR for babies.
Kendall toy testing for >2yo.
Tympanometry tests compliance of tympanic membrane, not hearing acuity. Primarily used to check for glue ear.

89
Q
During intermittent positive pressure ventilation, increasing which of the following (in isolation) would be least likely to improve carbon dioxide removal?
A. Delivered tidal volume.
B. Inspiratory time.
C. Peak inspiratory pressure.
D. Positive end-expiratory pressure.
E. Ventilator rate.
A

D - PEEP

Uptodate

●Ventilation – CO2 clearance is largely determined by minute ventilation, which in turn is determined by the frequency and size of breaths (ie, respiratory rate [RR] and tidal volume [Tv]). In volume-targeted modes, Tv is a setting and is controlled by the ventilator. In pressure-limited modes, Tv is determined by other settings (inspiratory pressure and inspiratory time [Ti]) and will vary depending upon lung compliance, with compliant lungs having higher Tvs at any given inspiratory driving pressure compared with less compliant lungs.

●Oxygenation – Oxygenation is primarily determined by the fraction of inspired oxygen (FiO2) and the mean airway pressure (MAP). In CMV, MAP is largely determined by the set PEEP since the expiratory phase usually predominates the respiratory cycle. Other settings (ie, Tv, inspiratory pressure, and Ti) influence MAP to a lesser extent.

90
Q
Which one of the following capsular polysaccharides of Neisseria meningitidis is least immunogenic?
A. A.
B. B.
C. C.
D. W-135.
E. Y.
A

B - B

91
Q

A 10-year-old boy presents with a history of recurrent palpitations, precipitated by strenuous exercise, over
the last 12 months. On the day of presentation he had the sudden onset of a rapid heart rate while playing
cricket at school. On arrival at triage he has a pulse rate of 240/minute, a respiratory rate of 29/minute and a
blood pressure of 80/50 mmHg. He is pale but alert and after a large vomit at triage his rapid heart rate
spontaneously resolves. His 12-lead electrocardiogram (ECG) is shown above.
An echocardiogram is most likely to show which of the following?
A. Dilated cardiomyopathy.
B. Ebstein anomaly.
C. l-Transposition of the great arteries (corrected transposition).
D. Mitral valve prolapse.
E. Normal cardiac anatomy.

A

ECG shows WPW with short PR, wide QRS, T wave inversion/opposite to QRS.

E - normal anatomy

92
Q

Which one of the following cells of the immune system is most reliant on its release of mediators into the
extracellular fluid surrounding the organism, in order to carry out its role in eradication of foreign organisms?
A. Cytotoxic T lymphocytes.
B. Eosinophils.
C. Macrophages.
D. Natural killer cells.
E. Neutrophils.

A

B - eosinophils

Special role in allergic disease and defence against parasites, and removal of fibrin formed during inflammation.

93
Q

The risk of externalising (antisocial or under-controlled behaviour) among school-aged girls is highest in
families with which one of the following demographic characteristics?
A. Divorced parents.
B. Low income.
C. Parents together, both unemployed.
D. Rural abode.
E. Unemployed single parent.

A

E - unemployed single parent

94
Q

A five-year-old boy with a diagnosis of intractable epilepsy is prescribed a new anticonvulsant. About one
month later, he presents with a 24-hour history of abdominal pain, vomiting and low-grade fever. The pain
worsens over the subsequent 24 hours and he develops intractable vomiting.
On examination he is dehydrated, mildly jaundiced and febrile. He has tenderness in the epigastric area. A
serum amylase is elevated. He is diagnosed with pancreatitis.
Which of the following anticonvulsants is most likely to be associated with this clinical scenario?
A. Carbamazepine.
B. Lamotrigine.
C. Phenytoin.
D. Sodium valproate.
E. Topiramate.

A

D - valproate

See other card

95
Q
Which of the following is the most appropriate antibiotic to treat a child with pneumococcal meningitis caused
by a strain of Streptococcus pneumoniae with intermediate resistance to penicillin (minimum inhibitory
concentration (MIC) 1 mg/L)?
A. Ceftriaxone.
B. Cotrimoxazole.
C. Erythromycin.
D. Penicillin G.
E. Vancomycin.
A

E - vancomycin

Strep pneumo resistance is mediated through altered penicillin binding proteins.

96
Q

The use of high frequency oscillation compared with conventional mechanical ventilation has been
demonstrated to be of most benefit in reducing the incidence of:
A. chronic lung disease.
B. intraventricular haemorrhage.
C. mortality.
D. pneumothorax.
E. retinopathy of prematurity.

A

A - CLD

97
Q

In a patient with a ventricular septal defect (VSD), which of the following clinical signs best correlates with a
pulmonary to systemic blood flow ratio (Qp/Qs) of greater than 2:1?
A. Apical mid-diastolic murmur.
B. Loud ejection systolic murmur.
C. Loud pulmonary component of the second heart sound.
D. Prominent systolic thrill.
E. Reversed splitting of the second heart sound.

A

A - apical mid-diastolic murmur

Parks

An apical diastolic rumble is present with a moderate-large shunt, caused by an increased flow through the mitral valve during diastole.

98
Q
Specific lung compliance is best standardised by measuring lung compliance at which of the following lung
volumes?
A. Expiratory reserve volume.
B. Functional residual capacity.
C. Inspiratory reserve volume.
D. Residual volume.
E. Total lung capacity.
A

B - functional residual capacity

99
Q
A six-year-old child has a syncopal episode in association with an airway infection. An electrocardiogram
(ECG) shows a QTc of 500 milliseconds.
Which of the following medications is least likely to exacerbate the prolonged QT interval?
A. Cisapride.
B. Dexamphetamine.
C. Erythromycin.
D. Imipramine.
E. Ketoconazole.
A

B - dexamphetamine

100
Q

A 10-month-old boy has a longstanding history of failure to thrive, vomiting and recurrent dehydration.
Which one of the following is most consistent with a diagnosis of distal renal tubular acidosis?

Serum sodium Serum potassium Urinary calcium
A. high high normal
B. high low low
C. low high low
D. normal high normal
E. normal low high
A

E - Na normal, K low, Ca high

K and H move in opposite directions, hence giving sodium bicarb for hyperkalaemia. Acidosis therefore causes hypokalaemia. Acidosis also increases bone resorption, to mobilise buffers from bony matrix, therefore increasing calcium -> urinary calcium elevated.

101
Q
Which of the following is more characteristic of streptococcal toxic shock syndrome than of staphylococcal
toxic shock syndrome?
A. Conjunctivitis.
B. Encephalopathy.
C. Generalised erythema.
D. Intractable hypotension.
E. Septicaemia.
A

E - septicaemia

Uptodate

Staphylococcal toxic shock syndrome (TSS) is a clinical illness characterized by rapid onset of fever, rash, hypotension, and multiorgan system involvement. Manifestations of multiorgan system involvement include gastrointestinal symptoms (abdominal pain, vomiting, and/or watery diarrhea), myalgias, mucous membrane (vaginal, oropharyngeal, or conjunctival) involvement, renal involvement, hepatic involvement, thrombocytopenia, and neurologic symptoms (headache, somnolence, confusion, irritability, agitation, hallucinations). Other manifestations include cyanosis and edema of the extremities. Detection of S. aureus in culture is not required for the diagnosis of staphylococcal TSS. S. aureus is recovered from blood cultures in approximately 5 percent of cases.

Streptococcal TSS – Streptococcal TSS typically presents with pain that precedes physical findings of infection. At the site of minor trauma (such as a bruise, strained muscle, or sprained ankle), patients may develop deep infection such as necrotizing fasciitis or myonecrosis, often with no visible break in the skin. The diagnosis of streptococcal TSS consists of hypotension and isolation of streptococci from a normally sterile site, together with two or more findings including renal dysfunction, coagulopathy liver dysfunction, acute respiratory distress syndrome, rash, and/or soft tissue necrosis. Blood cultures are positive for GAS in approximately 60 percent of cases.

●Toxic shock syndrome – Streptococcal and staphylococcal toxic shock syndromes are toxin-mediated systemic bacterial infections that can mimic SSSS. The inciting infections are usually a localized abscess, pyomyositis, or superinfection of a retained foreign body, such as tampons or surgical packing. Typical findings include widespread, blanchable, cutaneous erythema; conjunctival injection; “strawberry tongue;” fever; and hypotension. Unlike SSSS, there is a lack of periorificial crusting on the face, absent bullae or desquamation, and a negative Nikolsky sign. Acral skin peeling or desquamation can occur but usually appears several days after the onset of symptoms

102
Q

A nine-year-old boy presents with recurrent episodes of obstructive dysphagia. Over the past year he has
required endoscopic removal of impacted food in the upper oesophagus on several occasions. A barium
meal is normal.
Which of the following is the most likely diagnosis?
A. Achalasia.
B. Barrett’s oesophagus.
C. Eosinophilic oesophagitis.
D. Globus hystericus.
E. Mediastinal tumour.

A

C - EoE

MRCPCH

EoE is an important differential diagnosis of GOR that presents with similar symptoms but fails to respond to conventional acid blockade therapy. In older children, dysphagia and food impaction can occur. It is more commonly seen in patients with atopy or family history of atopic disease. Diagnosis is based on characteristic endoscopic findings/histology. Skin prick testing and RAST are not generally informative. Treatments include dietary elimination, steroids, immunosuppressants.

Natural history is of relapse and remission and chronicity.

103
Q
When assessing an adolescent, whose history is likely to have the greatest sensitivity for the detection of
depressive symptoms?
A. Friends.
B. Siblings.
C. The adolescent.
D. The classroom teacher.
E. The mother.
A

C - the adolescent

104
Q

The carrier frequency for cystic fibrosis in the Caucasian population in Australia is approximately 4%. The
∆F508 mutation accounts for 75% of all mutations among carriers in this population.
A girl is diagnosed with cystic fibrosis. DNA studies identify one ∆F508 mutation but fail to identify a
mutation in the other allele. DNA studies of her unaffected brother are normal.
What is the risk of the brother being a carrier?
A. <1%.
B. 4%.
C. 25%.
D. 50%.
E. 66%.

A

D - 50%

Hasn’t inherited delta F508 but 50% chance of inheriting the other allele.

105
Q

A 13-year-old girl presents to the emergency department after ingesting an unknown quantity of a medication
found in the bathroom cupboard. On arrival she is agitated and confused, flushed and has dry mucous
membranes. Her temperature is 38.5°C and her heart rate is 140/minute with a respiratory rate of 35/minute.
She has dilated pupils which react poorly to light.
Ingestion of which of the following best explains the clinical presentation?
A. Codeine phosphate.
B. Eucalyptus oil.
C. Imipramine.
D. Theophylline.
E. Verapamil.

A

C - imipramine (a TCA, also anticholinergic)

Anticholinergic = agitation, dry, mydriasis (Dilated pupils), fever, hallucination
TCA = convulsions, coma, QT prolongation
Codeine = opiate = resp depression, miosis (constricted)
Verapamil = CCB = heart block, hypotension, insulin resistance and hypoglycaemia
106
Q

A three-month-old girl presents with a facial lesion (shown above - haemangioma). She is commenced on oral prednisolone at a dose of 2 mg/kg/day. At review, two weeks after starting treatment, the lesion has grown and is now impinging significantly on her visual axis.

The most appropriate next step in her management is to:
A. administer interferon alfa subcutaneously.
B. cease the oral corticosteroids.
C. excise the lesion.
D. increase the dose of oral corticosteroids.
E. inject corticosteroids into the lesion.

A

A - administer interferon alfa subcut - out of date question.

Uptodate

High-risk/complicated hemangiomas that require treatment include large hemangiomas at increased risk of scarring or disfigurement (eg, large or segmental hemangiomas, facial hemangiomas), life-threatening hemangiomas (eg, airway hemangiomas), hemangiomas carrying functional risks (eg, periocular hemangiomas), and hemangioma ulceration that does not respond to standard wound care.

Propranolol — Propranolol, a nonselective beta blocker, is the first-line agent for hemangiomas with the potential to impair function or cause permanent disfigurement, if there are no cardiac or neurovascular concerns.

Systemic corticosteroids — Treatment with systemic corticosteroids for complicated hemangiomas has been largely supplanted by systemic beta blockers. However, systemic corticosteroids remain a treatment option for patients with complicated hemangiomas for whom treatment with beta blockers is contraindicated.

Vincristine and interferon alfa — Vincristine and interferon alfa are alternative systemic agents for the treatment of complicated hemangiomas but are rarely used since the advent of propranolol.

Surgical therapies — When a hemangioma poses primarily cosmetic concerns, therapeutic intervention must be tailored on an individual basis. In addition to systemic, topical, and intralesional medications, therapeutic options include laser therapy and surgery. Cautery, irradiation, and cryotherapy may produce more scarring than untreated lesions and are thus not generally recommended.

107
Q

A three-year-old boy with beta thalassaemia presents with three days of fever to 39.5°C, and 24 hours of
watery diarrhoea, colicky abdominal pain, poor oral intake and decreased urine output. He has blood
transfusions every four weeks and receives a desferrioxamine transfusion six nights a week.

On examination he is febrile (39.4°C) and alert but looks ill. He is not clinically dehydrated. His pulse rate is
165/minute, with a capillary refill time of 3 seconds. His respiratory rate is 32/minute. His abdomen is
distended and generally slightly tender. Bowel sounds are present. His liver and spleen are both palpable
2-3 cm below the costal margin.

Infection with which of the following is the most likely cause of his illness?
A. Escherichia coli.
B. Rotavirus.
C. Salmonella enteritidis.
D. Streptococcus pneumoniae.
E. Yersinia enterocolitica.
A

E - Yersinia enterocolitica.

Wikipedia

Siderophilic bacteria are bacteria that require or are facilitated by free iron. They may include Vibrio vulnificus, Listeria monocytogenes, Yersinia enterocolica, Salmonella enterica (serotype Typhimurium), Klebsiella pneumoniae and Escherichia coli. One possible symptom of haemochromatosis is susceptibility to infections from these species.

108
Q

An 18-month-old girl presents with severe pallor associated with irritability and anorexia. A full blood count is performed and is as follows:

haemoglobin (Hb) 28 g/L [105-140]
mean corpuscular volume (MCV) 90 fL [70-86]
white cell count (WCC) 3.9 x 109/L [6.0-17.5]
platelet count 135 x 109/L [150-400]
reticulocytes 0.4% [<2.0]
A bone marrow aspirate (A) and blood film (B) are shown below.

Which of the following is the most likely diagnosis?
A. Acute lymphoblastic leukaemia.
B. Autoimmune haemolytic anaemia.
C. Diamond-Blackfan anaemia.
D. Human parvovirus B19 infection.
E. Transient erythroblastopenia of childhood.

A

C - Diamon Blackfan anaemia

Congenital erythroid aplasia. Generally presents in infancy, most <12 months old.
Normochromic macrocytic reticulocytopenic anaemia, associated with congenital malformations (50% and classically thumbs), increased risk of malignancy (AML, MDS), and endocrine dysfunction (e.g. short stature).
Ribosomopathy -> activates TP53 pathway.
BMAT -> normal cellularity with reduced erythroid precursors.

Differentials: TEC (usually older and preceded by viral illness), other marrow failure syndromes.

109
Q

A 12-year-old boy was born in Thailand and was fully immunised, including neonatal BCG (bacille Calmette-
Guérin). The family migrated when the boy was four years old. The whole family was well. They had re-
visited Thailand for the first time one year previously, staying in a city with their family. They had bought a
kitten six months earlier which would often scratch the boy when he played with it.

Over a three-month period the boy developed a lump in his neck, which gradually enlarged. It was painless
and not discoloured. He had no fever or other systemic upset and had not lost weight.

On examination the boy is well and afebrile. He has a 5 cm rubbery oval-shaped mass palpable in the right
posterior cervical triangle and no other positive findings on clinical examination.

Which of the following is the most likely diagnosis?
A. Atypical mycobacterial infection.
B. Cat scratch disease.
C. Cytomegalovirus infection.
D. Toxoplasmosis.
E. Tuberculosis.
A

E - tuberculosis

110
Q

A 14-year-old boy presents to the emergency department with a painful right eye after being hit in the eye with a stone. On examination, his right pupil is slightly dilated and reacts sluggishly to light, he has reduced visual acuity on the right, and his extraocular eye movements are normal with no pain. The appearance of his right eye on examination is shown below.

Hyphema - blood in the anterior chamber of the eye.

Which of the following is the most likely acute complication?
A. Glaucoma.
B. Hypopyon.
C. Retinal artery occlusion.
D. Retinal detachment.
E. Spontaneous re-bleed.
A

E - spontaneous rebleed

Uptodate

Traumatic hyphema, or blood in the anterior chamber, is a common complication of blunt or penetrating injury to the eye and can result in permanent vision loss. The goals of initial assessment include recognition and characterization of the hyphema and identification of associated orbital and ocular injuries. If ruptured globe is suspected, then emergent consultation with an ophthalmologist is critical. In addition, optimal outcome following a hyphema depends on early ophthalmologic intervention focused on prevention of rebleeding and avoidance of intraocular hypertension. In most instances, patients recover with normal vision. Vision loss is more likely in patients with large hyphemas, sickle hemoglobinopathy/trait, or bleeding disorders.

●Rebleeding – Secondary hemorrhages usually result in hyphemas that are larger than the initial injury and can markedly increase the risk of vision loss.

111
Q

A two-year-old girl is known to have doubly committed (supracristal, conal) ventricular septal defect. Over
the six months prior to review her mother reports a significant deterioration in exercise tolerance.
The most likely aetiology of the progressive fatigue is:
A. aortic regurgitation.
B. bacterial endocarditis.
C. hypocalcaemia
D. increasing left to right shunt.
E. mitral regurgitation.

A

A - AR

112
Q

A 10-year-old boy presents with a 16-month history of recurrent eye blinking, shoulder shrugging and rapid
movements of his eyes. In addition, he has had periods of recurrent throat clearing, sniffing, barking and
grunting. The events tend to wax and wane but are most prominent at school or when he is under stress.
By report he has had longstanding issues with attention and hyperactivity in the classroom. He is diagnosed
with Tourette syndrome.

He is being teased at school and is now refusing to attend school and a decision is made to institute therapy.

Which of the following therapies is most likely to be beneficial in this child?
A. Behavioural therapy.
B. Clonidine.
C. Family therapy.
D. Haloperidol.
E. Methylphenidate.
A

D - haloperidol

113
Q

A term male infant weighing 2900 g at birth was found to have a cleft palate and moderate micrognathia. No
airway obstruction was noted. He established enteral feeds with a cleft palate bottle and was discharged
home at seven days of age. He was seen by a paediatrician at three weeks of age. His mother related that he was taking over 40 minutes to take each feed and vomited after most feeds. His weight was 2400 g.

The most appropriate initial step in management would be:
A. fundoplication and gastrostomy.
B. increase the caloric density of the feeds.
C. insert a nasopharyngeal airway.
D. nasogastric feeding.
E. thicken the feeds and treat with ranitidine.

A

C - insert a nasopharyngeal airway

MRCPCH

Cleft lip and palate is often diagnosed antenatally on US scanning. It can be associated with Pierre Robin sequence (as in this case, micrognathia and flossoptosis), other dysmorphic syndromes (Stickler, Edwards), or in isolation. Certain antiepileptic medications are associated with increased risk.

Issues

  • Cosmetic: cleft lip is repaired early at 2-3mo
  • Feeding: special teats, palatal plate, may be associated oropharyngeal incoordination and aspiration of feeds, cleft palate surgery is usually performed at 1yo
  • Speech and hearing: frequent AOM, glue ear
  • Airway obstruction: if d/t Pierre Robin sequence some infants require a nasopharyngeal tube to maintain patency of the upper airway
114
Q

A 16-year-old girl with type 1 (insulin-dependent) diabetes mellitus attends for a routine review. She attends
with her mother but asks to be seen by herself and then describes significant conflict with both parents. She
has discussed the possibility of leaving home with a school counsellor whom she has seen several times
over the past month. She acknowledges being variably compliant in taking her insulin and her glucometer
readings are high.
On examination you notice several superficial lacerations on her left forearm. She informs you that she has
frequent thoughts about killing herself but is adamant that she doesn’t want her parents informed.
Which of the following is the most appropriate course of action?
A. Admit her to hospital, informing her parents that you are doing so because of adolescent issues.
B. Carefully discuss her suicidal ideation, have her sign a contract that she will not harm herself
and review her the next day.
C. Contact the school counsellor and arrange for daily counselling to support the patient until your
review the following week.
D. Invite her mother to join the interview and inform the mother that you feel the patient is at risk.
E. Respect the patient’s wish for confidentiality, adjust her insulin and review her the next day.

A

A - admit her to hospital, informing her parents that you are doing so because of adolescent issues

115
Q

A 13-year-old boy presents with a two-week history of left-sided hip and knee pain following a cricket game.
The pain has worsened over the preceding two days following a fall from his bike.
On examination, the boy is afebrile and is not distressed, but is unable to weight bear on his left leg. He has
limited movement of his left hip in all directions secondary to pain and there is no evidence of swelling,
bruising or deformity over the hip or knee. An X-ray of his pelvis is shown below.

?SUFE

Which of the following is the most appropriate next diagnostic step?
A. Bone scan.
B. Frog-leg lateral X-ray of left hip.
C. Full blood count (FBC) and erythrocyte sedimentation rate (ESR).
D. Left knee X-ray.
E. Ultrasound of left hip.

A

B - frog leg x-ray

RCH guideline

Pelvis AP or frog leg view is useful for identifying SUFE, DDH (>6 mo), Perthes disease and common pelvic avulsions.

116
Q

A 15-year-old girl recently had unprotected sexual intercourse with a man known to be HIV (human
immunodeficiency virus)-positive.
Which of the following tests is most helpful in establishing whether she has been infected with HIV?
A. HIV antibodies measured by enzyme-linked immunosorbent assay (ELISA).
B. HIV antibodies measured by Western blot.
C. P24 antigen.
D. Qualitative polymerase chain reaction (PCR) for HIV DNA.
E. Quantitative HIV RNA viral load by reverse transcriptase (RT) PCR.

A

D - qualitative PCR for HIV DNA

117
Q

An 18-month-old girl presents with vaginal bleeding. Examination reveals Tanner stage 3 breast
development and no pubic hair. There is a pigmented lesion on her buttock as seen in the photograph
below.

2x hyperpigmented lesions, one over left buttock one higher up on mid spine.

Which of the following is the most likely finding on investigation?
A. Delayed bone age.
B. Elevated luteinising hormone (LH) and follicle-stimulating hormone (FSH).
C. Elevated serum testosterone.
D. Large ovarian cyst on pelvic ultrasound.
E. Pituitary tumour on cerebral magnetic resonance imaging (MRI) scan.

A

D - large ovarian cyst on pelvic ultrasound

?McCune Albright syndrome. Uptodate:

McCune-Albright syndrome — McCune-Albright syndrome is a rare disorder defined as the triad of peripheral precocious puberty, irregular café-au-lait (“coast of Maine”) skin pigmentation, and fibrous dysplasia of bone. MAS should be considered in females with recurrent formation of follicular cysts and cyclic menses. The skin manifestations and bone lesions may increase over time. In females presenting with vaginal bleeding, the ovarian enlargement has often been mistaken for an ovarian tumor, leading to unnecessary oophorectomy. Females presenting with premature vaginal bleeding should therefore be evaluated for features of MAS to avoid this potential mistake.

The clinical phenotype varies markedly, depending on which tissues are affected by the mutation, but precocious puberty is the most commonly reported manifestation. As in other forms of peripheral precocity, the sequence of pubertal progression may be abnormal, in that vaginal bleeding often precedes significant breast development. Prolonged exposure to elevated levels of sex steroids may cause accelerated growth, advanced skeletal maturation, and compromised adult height. Although the precocious puberty is typically peripheral precocity, a secondary component of CPP may develop because of sex steroid withdrawal leading to activation of the hypothalamic-pituitary-gonadal axis. In males with MAS, while sexual precocity is less common, there is a high prevalence of testicular pathology on ultrasound, including hyper- and hypoechoic lesions (most likely representing areas of Leydig cell hyperplasia), microlithiasis, and focal calcifications.

Patients with MAS have a somatic (postzygotic) mutation of the alpha subunit of GNAS, which encodes the Gs protein that activates adenylyl cyclase. This mutation leads to continued stimulation of endocrine function, including precocious puberty, thyrotoxicosis, growth hormone excess (gigantism or acromegaly), Cushing syndrome, and renal phosphate wasting (hypophosphatemic rickets) in various combinations. Mutations can be found in other nonendocrine organs (liver and heart) resulting in cholestasis and/or hepatitis, intestinal polyps, and cardiac arrhythmias, respectively. A heightened risk of malignancy has also been reported. Germline occurrences of this mutation would presumably be lethal

118
Q

A four-year-old girl with Alagille syndrome (intrahepatic hypoplasia of bile ducts) and long-standing
cholestatic jaundice presents with a broad-based ataxic gait. On examination her tendon reflexes are
absent.
Which of the following is the most likely cause?
A. Folate deficiency.
B. Hypothyroidism.
C. Posterior fossa tumour.
D. Vitamin B12 deficiency.
E. Vitamin E deficiency.

A

E - vit E deficiency

Bile deficiency -> impaired fat soluble vitamin absorption.

Vitamin E deficiency: neuropathy, ataxia, retinal degeneration, haemolytic anaemia.

119
Q

A five-year-old boy presents with a 10-day history of purulent nasal discharge associated with fever and
intermittent cough. His symptoms have failed to respond to a seven-day course of oral amoxycillin. He has
had three similar episodes in the past year.
The most appropriate initial investigation is:
A. chest X-ray.
B. computed tomography (CT) scan of the facial sinuses.
C. culture of the nasal discharge with determination of antibiotic sensitivities.
D. plain X-ray of the facial sinuses.
E. serum immunoglobulin levels.

A

B - CT of the facial sinuses

120
Q

A 12-year-old girl has been absent from school for the past six weeks, following sporadic attendance since
the start of the year. She describes feeling panicky every time she approaches the school gates. She
began secondary school this year and says that she doesn’t like the teachers or other pupils. She enjoyed
primary school, achieved good academic levels and had two close friends, one of whom has gone to a
different secondary school.
She is an only child and her mother presents as anxious and ineffectual, and recalls some difficulties getting
her daughter to school at the beginning of each primary school year. Her father had more success by driving
her to school in previous years, but over the last few weeks his patience has been exhausted and believes
her year-level coordinator has not given much support. He raises the possibility of a fresh start at another
local school. The girl wishes to enrol in the same school as her friend.

Which of the following is the most appropriate recommendation?
A. Change schools.
B. Cognitive-behavioural therapy.
C. Commence a selective serotonin reuptake inhibitor (SSRI).
D. Distance education.
E. Parental counselling.

A

B - CBT

121
Q

An 18-month-old boy presents with a history of two episodes of having suddenly collapsed to the ground
while running. He was found to be pallid on both occasions, with no spontaneous movements and no
apparent respirations. On both occasions his parents gave him cardio-pulmonary resuscitation. Normal
respiration was restored after a few minutes, and he was tired for some hours after each event. After the
second occasion, he was admitted to hospital for observation and investigation. He has a normal
developmental history and has never had any serious illnesses recognised. There is no family history of
epilepsy.
This clinical presentation would be most consistent with:
A. atonic seizures.
B. breath holding episodes.
C. cardiac arrhythmia.
D. exercise-induced anaphylaxis.
E. hypoglycaemia.

A

C - cardiac arrhythmia

Exercise induced syncope - need to think about cardiac causes.

122
Q

An eight-year-old boy with achondroplasia is referred for evaluation of snoring. On examination he has large
tonsils. An overnight polysomnogram demonstrates five episodes per hour of non-obstructive oxygen
desaturation to less than 90%.

The most appropriate next step is:
A. adenotonsillectomy.
B. computed tomography (CT) scan of chest.
C. lateral airways X-ray of neck.
D. lung function testing.
E. magnetic resonance imaging (MRI) scan of brainstem and upper cervical cord.

A

E - MRI brainstem and cervical cord

Stem states non-obstructive desaturation, so not related to OSA/tonsils (answer A which I chose). Essentially describing central apnoea.

Uptodate complications of achondroplasia

●Sleep-disordered breathing and sleep apnea – Midface retrusion in conjunction with adenoid and tonsil enlargement can reduce the airway space, leading to obstructive sleep apnea (OSA) and sleep-disordered breathing. OSA is clinically manifested by loud snoring, breath holding during sleep (apnea), poor sleep, poor school performance, and behavioral problems in some cases. Sleep apnea can also be central in patients with cervicomedullary compression.

●Cervicomedullary compression – One area of controversy in achondroplasia is the prevalence and management of cervicomedullary compression due to narrowing of the foramen magnum. It is estimated that 5 to 10 percent of patients with achondroplasia develop true cervical medullary stenosis even though some degree of narrowing occurs in many more. Cervicomedullary compression is associated with significant morbidity and mortality, including an increased risk of sudden infant death. The maximum anatomical narrowing occurs at approximately 12 months of age. Thus, all infants should have computed tomography (CT) or MRI of the cervical medullary junction for surveillance around that time.

Signs of narrowing of the cervical medullary junction should be closely followed, but rushing to surgical intervention should be avoided in the absence of other clinical findings. Warning signs for compression include motor delay that is more severe, persists in time, or is associated with an abnormal neurologic exam. Other concerning findings include clonus, hyperreflexia of the lower limbs, central apnea/hypopnea, and a rapidly growing head circumference. A thorough evaluation should be performed to rule out cervicomedullary compression, and urgent referral to a neurosurgeon should be initiated if any of these findings are present. It is best if these evaluations and surgical decisions involve a multidisciplinary team including geneticists, orthopedists, neurologists, and neurosurgeons.

This evaluation includes a complete neurologic assessment, CT scan of the head with foramen magnum measurements, cerebrospinal fluid (CSF) flow studies of the cervical spine done with MRI with flexion-extension images and sleep studies to assess sleep apnea. Most groups do not recommend routine CT or MRI studies.

123
Q

A nine-month-old boy presents after a three-minute tonic-clonic seizure, associated with a fever of 39°C. He
is fully immunised. The infant has a number of small ulcers on the tip of the tongue and on the gingival
mucosa of the lower incisors. The following day the infant develops weakness of his left leg.

The infant is most likely to be infected with:
A. cytomegalovirus (CMV).
B. enterovirus 71.
C. Epstein-Barr virus (EBV).
D. human herpes virus 6 (HHV-6).
E. poliovirus type 3.
A

B - enterovirus 71. Febrile seizure, oral lesions, weakness.

HHV-6 is the most common cause of febrile seizures but shouldn’t cause neurology. Enterovirus 71 causes HFMD and acute paresis/flaccid myelitis.

Uptodate

Infection — Viral infections are commonly identified in association with febrile seizures, whereas bacterial infections are infrequent. Febrile seizures are not thought to be viral specific, but rather dependent upon the degree of temperature elevation. Viral infections associated with high fever, such as human herpesvirus 6 (HHV-6) and influenza, appear to pose the highest risk.

HHV-6 is the virus most frequently associated with febrile seizures in the United States.

Certain enteroviruses (ie, polioviruses, enterovirus D68, enterovirus A71) preferentially target motor nuclei within the brainstem and spinal cord, causing acute paresis of cranial and spinal nerves. The syndrome of acute motor neuron weakness (also known as acute flaccid paralysis or acute flaccid myelitis [AFM], among other terms) has been reported with many enterovirus serotypes, but only a small number are associated with endemic and epidemic paralysis: poliovirus types 1, 2, and 3, enterovirus D68, and enterovirus A71.

Enterovirus 71-associated acute flaccid myelitis is distinguished by a common association with concurrent HFMD and occasional myoclonus, ataxia, and autonomic instability that may represent brainstem encephalitis. In young children, a severe form of enterovirus 71 brainstem encephalitis accompanied by noncardiogenic pulmonary edema and leading to a rapidly fatal course may occur.

124
Q
A male infant is born at term, weighing 2200 g. He is noted to have profound hypotonia and absent reflexes.
He has moderately severe hepatomegaly. His facial appearance is shown above.
The most likely diagnosis is:
A. myotonic dystrophy.
B. Prader-Willi syndrome.
C. spinal muscular atrophy.
D. trisomy 21.
E. Zellweger syndrome.
A

E - Zellweger syndrome

MRCPCH

Presents in neonatal period. High forehead, patent fontanelles. Severe hypotonia and poor sucking/swallowing. Very poor subsequent neurological development. Often a/w cerebral gyral abnormalities.

Peroxisomal disorder. Spectrum with Zellweger, neonatal adrenoleukodystrophy, Refsum disease.

Peroxisome function: synthesis of plasmalogens, cholesterol, bile acids, beta-oxidation of VLCFA and breakdown of phytanic acid (vitamin A) and flyoxylate.

AR inheritance. First line investigation is VLCFA.

125
Q

A six-month-old girl presents with a short history of fever (38°C) and exacerbation of her facial eczema, as
shown in the photograph above. She has a strong family history of asthma.
Initial treatment with oral flucloxacillin and topical 1% hydrocortisone ointment shows no improvement.
Which of the following is most likely to improve her condition?
A. Intravenous aciclovir.
B. Intravenous amoxycillin/clavulanic acid.
C. Intravenous flucloxacillin.
D. More potent topical steroids.
E. Wet saline dressings.

A

A - IV aciclovir.

Fluclox ineffective. Bioavailability ~60%.

126
Q

You interview a 15-year-old girl with a history of child sexual abuse. She often fights with her mother and
stepfather and they describe a worsening in her behaviour over the past six months. She has outbursts of
anger after slight provocation, is sensitive to loud noises and stays up after they have gone to bed. She is
performing badly at school. At times she seems normal and can enjoy going out, but her irritability has left
her with few friends.
When interviewed by herself, the girl is alert and shows no features of thought disorder. She startles at
noises from outside the consulting room. She describes difficulties getting to sleep and concentrating at
school but denies any drug use. She then tells you that she frequently hears male voices at home and
school making derogatory comments about her and telling her to cut herself. The voices, which she doesn’t
recognise, have been occurring for months and often follow fights with her parents.
Her presentation is most consistent with a primary diagnosis of:
A. depression.
B. dissociative disorder.
C. post-traumatic stress disorder.
D. schizophrenia.
E. substance abuse.

A

C - PTSD

127
Q

A 25-week gestation baby girl, birth weight 720 g, was ventilated from birth. She received surfactant,
intravenous antibiotics for five days, and was given parenteral nutrition because of feed intolerance. At 28
days of age, she became pale and lethargic. Her blood count was:
haemoglobin (Hb) 152 g/L [140-180]
white cell count (WCC) 14.2 x 109/L [5.0-15.0]
platelet count 92 x 109/L [150-400]

You are called to say that blood cultures are growing a yeast, probably Candida. Chest X-ray is unchanged,
urine and cerebrospinal fluid (CSF) microscopy is normal, and renal ultrasound is normal.
Which of the following is the most appropriate treatment?
A. Amphotericin B.
B. Fluconazole.
C. Flucytosine.
D. Itraconazole.
E. Miconazole.

A

SYSTEMIC ANTIFUNGAL AGENTS
The four different classes of antifungal agents used to treat invasive neonatal candidiasis are:
●Polyenes – Polyenes include amphotericin B deoxycholate, which is the preferred antifungal agent for most invasive neonatal Candida infections.
●Triazoles – The most commonly used triazole in neonates is fluconazole.
●Nucleoside analogues – The most commonly used nucleoside analogue in neonates is flucytosine. It is sometimes used in combination with amphotericin B in neonates with central nervous system (CNS) infections.
●Echinocandins – Echinocandins include caspofungin, anidulafungin, and micafungin. These drugs are not routinely used for neonatal Candida infections.

Amphotericin B — Amphotericin B deoxycholate is the preferred drug for treatment of most systemic neonatal Candida infections. Most Candida species are susceptible to amphotericin B, and this drug is efficacious and well tolerated in neonates.

Fluconazole — Fluconazole, a first-generation triazole, is an alternative to amphotericin B in some infants, if the isolate is susceptible. Fluconazole has several advantages:
●Excellent therapeutic levels throughout the body including the CSF
●Excellent agent for the treatment of Candida UTI because it is excreted unchanged in high concentrations into the urine
●Excellent bioavailability when taken orally, potentially reducing the need for intravenous administration

The major disadvantage of this agent is the emergence of fluconazole-resistant Candida species. Thus, fluconazole should not be used as the agent for initial treatment of an infant with candidemia or suspected candidemia. It should be given only after identifying the Candida species and determining that the organism is sensitive to fluconazole.

128
Q

A two-month-old girl is admitted to hospital for treatment of a high fever and profuse diarrhoea. Intravenous
rehydration is commenced. Stool cultures reveal infection with Salmonella enteritidis. On day 3 of the
illness her fever persisted. She becomes increasingly irritable and has a prolonged generalized tonic-clonic
seizure on day 4.
Which of the following is the most likely cause of the seizure?
A. Hypernatraemia.
B. Hypocalcaemia.
C. Hypoglycaemia.
D. Hyponatraemia.
E. Meningitis.

A

E - meningitis

129
Q
The protein shared by the alternate and classical complement systems is:
A. C1.
B. C2.
C. C3.
D. C4b.
E. Properdin.
A

C - C3

Classical = antigen-antibody complexes, C1/2/4 then activate C3.
Alternate = direct activation by microbes (molecules on pathogen surface), direct activation of C3.
Lectin pathway = sugar residues and mannose, MBL, C2/4

130
Q

A two-year-old boy is brought into the emergency department unconscious following a house fire. His
carboxyhaemoglobin level is measured on presentation as 40%. Which of the following mechanisms
is most important in producing the toxic effects of this inhalation?
A. Direct toxic effect on myoglobin.
B. Irreversible binding of carbon monoxide to haemoglobin.
C. Irreversible binding of carbon monoxide to myelin.
D. Reversible binding of carbon monoxide to cytochrome A3.
E. Shifting of the oxygen dissociation curve to the right.

A

D - reversible binding of carbon monoxide to cytochrome A3.

Uptodate

Carbon monoxide (CO) is an odorless, tasteless, colorless, nonirritating gas formed by hydrocarbon combustion. The atmospheric concentration of CO is generally below 0.001 percent, but it may be higher in urban areas or enclosed environments. CO binds to hemoglobin with much greater affinity than oxygen, forming carboxyhemoglobin (COHb) and resulting in impaired oxygen transport and utilization. CO can also precipitate an inflammatory cascade that results in CNS lipid peroxidation and delayed neurologic sequelae.

Carbon monoxide (CO) diffuses rapidly across the pulmonary capillary membrane and binds to the iron moiety of heme (and other porphyrins) with approximately 240 times the affinity of oxygen. Nonsmokers may have up to 3 percent carboxyhemoglobin at baseline; smokers may have levels of 10 to 15 percent. Once CO binds to the heme moiety of hemoglobin, an allosteric change occurs that greatly diminishes the ability of the other three oxygen binding sites to off-load oxygen to peripheral tissues. This results in a deformation and leftward shift of the oxyhemoglobin dissociation curve, and compounds the impairment in tissue oxygen delivery.

CO also interferes with peripheral oxygen utilization. Approximately 10 to 15 percent of CO is extravascular and bound to molecules such as myoglobin, cytochromes, and NADPH reductase, resulting in impairment of oxidative phosphorylation at the mitochondrial level. CO also interferes with peripheral oxygen utilization by inactivating cytochrome oxidase in a manner similar to, but clinically less important than, cyanide.

The effects of CO on oxygen delivery and utilization, however, cannot account for the delayed neurologic sequelae (DNS) that may occur after CO poisoning. The mechanism of DNS is incompletely understood.

Carboxyhaemoglobin concentrations:

  • 10% not normally a/w symptoms
  • 10-30% headache, dyspnoea
  • 60% coma, convulsions, death

Treatment is with 100% oxygen. Hyperbaric oxygen works quicker, and is generally indicated with levels >25% (20% if pregnant), loss of consciousness, severe metabolic acidosis pH <7.1, evidence of end organ ischaemia (ECG changes, chest pain, altered mental status).

131
Q
Which of the following anticonvulsants is most likely to increase the half-life of topiramate?
A. Carbamazepine.
B. Phenobarbitone.
C. Phenytoin.
D. Primidone.
E. Sodium valproate.
A

E - valproate

Mnemonic

  • inhibitors: sickfaces.com
  • inducers: BS CRAP

Inhibitors (when in doubt, think of an azole):

  • S: sodium valproate
  • I: isoniazid
  • C: cimetidine
  • K: ketoconazole (lots of azoles)
  • F: fluconazole
  • A: alcohol (acute), amiodarone
  • C: chloramphenicol, ciprofloxacin
  • E: erythromycin
  • S: sulphonamides
  • C: cranberry juice, grapefruit juice
  • O: omeprazole
  • M: metronidazole

Inducers:

  • B: barbiturates
  • S: st johns wart, sulphonylureas
  • C: carbamazepine
  • R: rifampicin
  • A: alcohol (chronic)
  • P: pheonbarb, phenytoin
132
Q

In a 10-year-old with cystic fibrosis, which of the following lung parameters is the most reproducible?
A. Forced expiratory flow 25-75% (FEF25-75%).
B. Forced expiratory volume in 1 second (FEV1).
C. Forced vital capacity (FVC).
D. Intrathoracic gas volume (ITGV).
E. Residual volume to total lung capacity (RV:TLC) ratio.

A

B - FEV1

133
Q
Which one of the following steps in the design of a clinical trial of a new therapeutic agent most
reduces the chance of a type II error?
A. Blinded assessment of outcomes.
B. Intention to treat analysis.
C. Random allocation.
D. Sample size calculation.
E. Stratification of major risk factors.
A

D - sample size calculation

134
Q
In a child with a massive abdominal Burkitt lymphoma, the most useful treatment to prevent tumour
lysis syndrome is:
A. allopurinol.
B. frusemide.
C. mannitol.
D. prednisolone.
E. urate oxidase.
A

E - urate oxidase (rasburicase is a recombinant version of urate oxidase).

Wikipedia

MOA: Rasburicase catalyzes enzymatic oxidation of poorly soluble uric acid into an inactive and more soluble metabolite allantoin with carbon dioxide and hydrogen peroxide as byproducts in the chemical reaction.

Need to check G6PD as causes haemolysis.

135
Q
A resting human mast cell is shown above. Substances present in the mast cell granules include all of
the following except:
A. chondroitin sulphate.
B. heparin.
C. histamine.
D. interferon.
E. tryptase.
A

D - interferon

136
Q
Idiopathic ascending aortic dilatation would be most likely to accompany which of the following
syndromes/associations?
A. Down syndrome.
B. Noonan syndrome.
C. Turner syndrome.
D. VACTERL association.
E. Velocardiofacial syndrome.
A

C - turner syndrome

137
Q

A 1200 g 31-week gestation male infant is in good condition following a normal vaginal delivery. He is
nursed in room air in a single-walled incubator. At 24 hours of age his temperature per axilla is
35.5°C. His rectal temperature is 36.3°C. The incubator temperature is 34.1°C. The relative humidity
inside the incubator is 70%. The temperature of the wall of the incubator is 32°C. The nursery room
temperature is 23°C.
The most appropriate way of achieving a neutral thermal environment for this infant is to:
A. increase the incubator temperature.
B. increase the nursery environmental temperature.
C. increase the relative humidity of the incubator.
D. place a heat shield over the infant.
E. remove all draughts from the nursery.

A

D - place a heat shield over the infant

138
Q

Which one of the following is currently believed to be the most important contributor to the increase in
child body mass index (BMI) over the last 15 years?
A. Decreasing fitness.
B. Decreasing participation in sport.
C. Increasing calorie (energy) intake.
D. Increasing fat intake.
E. Increasing sedentary behaviour.

A

E - increasing sedentary behaviour

139
Q

A four-month-old boy is admitted to hospital for surgical management of his inguinal hernia.
Phenotypically he is a normal male. At surgery the hernial sac is found to contain uterine tissue. Both
testes are identified and appear normal.
Investigation shows him to have an XY karyotype.
Which one of the following is the most likely aetiology for these findings?
A. Absent Leydig cells.
B. Androgen insensitivity.
C. Deficiency of anti-Müllerian hormone.
D. 5α-reductase deficiency.
E. Gonadal dysgenesis.

A

C - deficiency of anti Mullerian hormone

140
Q
Which one of the following is least likely to lead to a progression of chronic renal failure?
A. Hyperlipidaemia.
B. Hypocalcaemia.
C. Metabolic acidosis.
D. Phosphate retention.
E. Proteinuria.
A

B - hypocalcaemia

141
Q

A new diagnostic test for a certain disease has been evaluated. Compared with the definitive
diagnostic standard, this test has a sensitivity of 100% and a specificity of 95%. The prevalence of the
disorder in the population to be tested is 0.1%.
What is the best estimate of the positive predictive value of the new test?
A. <1%.
B. 2%.
C. 5%.
D. 10%.
E. 25%.

A

B - 2%

Need to use prevalence to set up the table, i.e. 0.1% = 1 in 1000 people with the disease, so start there. True positive = 1, false negative = 0, hence sensitivity is 100%. True negatives = 950 with false positives 50 hence specificity is 95%.

This leaves PPV = 1/50 = 0.02.

142
Q

A rhythm strip from an electrocardiogram (ECG) is shown above. Which of the following is the most
likely cause for the abnormality displayed?

(Prolonged QT)

A. Hyperkalaemia.
B. Hyperthermia.
C. Hyperthyroidism.
D. Hypocalcaemia.
E. Uraemia.
A

D - hypocalcaemia.

Hyperkalaemia ECG: peaked T waves -> PR prolongation -> wide QRS VT -> terminal VF
Hypokalaemia: ECG on seen in severe hypokalaemia, U waves, T wave flattening, ST depression -> VT/VF, long QT and Torsades

Hypocalcaemia: ECG - prolongation of ST segment and QT interval -> VT
Hypercalcaemia: ECG - shortened QTc, Osborn or J wave

143
Q
Which research design can most conclusively demonstrate causality?
A. Case-control study.
B. Multiple-baseline intervention study.
C. Prospective cohort study.
D. Randomised controlled trial.
E. Retrospective cohort study.
A

D - RCT

144
Q

The daily energy requirement of a parenterally fed three-week-old neonate is estimated to be 100 kcal
(420 kJ) per kilogram body weight. Which of the following intravenous glucose and lipid solutions will
provide adequate daily energy for an infant of 4 kg body weight?
A. 400 mL of glucose 10% plus 20 mL of intravenous lipid 20%.
B. 400 mL of glucose 15% plus 30 mL of intravenous lipid 20%.
C. 400 mL of glucose 15% plus 40 mL of intravenous lipid 20%.
D. 400 mL of glucose 15% plus 50 mL of intravenous lipid 20%.
E. 400 mL of glucose 20% plus 60 mL of intravenous lipid 20%.

A

E - 400 mL of glucose 20% plus 60 mL of intravenous lipid 20%.

145
Q
Which of the following antipsychotic medications is most likely to cause the greatest weight increase
when prescribed to adolescents?
A. Haloperidol.
B. Olanzapine.
C. Pimozide.
D. Quetiapine.
E. Risperidone.
A

B - olanzapine

EPSE generally more common with typical/first gen antipsychotics (haloperidol).

Weight gain generally a common SE for atypical/second gen (olanzapine, quetiapine, risperidone). Olanzapaine is the worst.

146
Q

A haplotype is best defined as a group of alleles which:
A. are at a specific genomic locus for which an individual is heterozygous.
B. are closely linked together at a genomic locus.
C. are present on the same chromosome as a pathogenic mutation.
D. are specific to a particular ethnic group.
E. show reduced linkage disequilibrium.

A

B - closely linked together at a genomic locus

Google: A haplotype is a set of DNA variations, or polymorphisms, that tend to be inherited together. A haplotype can refer to a combination of alleles or to a set of single nucleotide polymorphisms (SNPs) found on the same chromosome.

147
Q

An eight-year-old boy is diagnosed as having a congenital antibody deficiency after being found to
have an IgG level of 0.50 g/L [4.95-16.56]. He weighs 25 kg. Six hours after infusion of 30 g of
intravenous immunoglobulin his IgG level is 9 g/L.
How many days later would you expect his IgG level to be at 4.5 g/L?
A. 3.
B. 7.
C. 14.
D. 28.
E. 56.

A

D - 28

Google: For total IgG, the half-life found was 25.8 days; for IgG1 it was 29.7 days; for IgG2 it was 26.9 days; and for IgG3 it was 15.7 days.

148
Q

Which of the following most accurately describes the known effects of supplementation of artificial milk
formulae with long chain polyunsaturated fatty acids (LCPUFA) in term infants?
A. A beneficial effect on bone maturation.
B. A beneficial effect on growth.
C. A beneficial effect on neurodevelopmental outcome.
D. A beneficial effect on visual maturation.
E. No additional benefit demonstrated.

A

E - no additional benefit demonstrated

149
Q
Which of the following is the chief determinant of intrathoracic airway resistance in normal children?
A. Airway length.
B. Alveolar pressure.
C. Lung elastic recoil.
D. Lung volume.
E. Number of conducting airways.
A

C - lung elastic recoil

150
Q
You wish to study the relative contributions of the home environment versus genetic influences in the
causation of literacy problems.
Which method used to study genetic influences would be most likely to answer this question?
A. Adoption studies.
B. Family aggregation studies.
C. Molecular genetic studies.
D. Pedigree analysis.
E. Twin studies.
A

A - adoption studies

151
Q

The ratio of systemic to pulmonary vascular resistances changes postnatally. What changes would be
expected to systemic vascular resistance (SVR) and pulmonary vascular resistance (PVR) one day
after normal delivery of a term infant in comparison to antenatal values?

SVR PVR
A. Decrease Decrease
B. Decrease No change
C. Increase Decrease
D. Increase No change
E. No change Decrease
A

C - increase SVR decrease PVR

152
Q

Among children and adolescents with significant behavioural disturbances, a poor prognosis is most
likely with which of the following?
A. Alcoholic or sociopathic father.
B. Greater severity of problem behaviour.
C. Lower age of presentation.
D. Maternal psychosocial problems.
E. Parental divorce.

A

B - greater severity of problem behaviour

153
Q
Which of the following interleukins has most influence on haematopoietic lymphoid growth and
differentiation?
A. Interleukin 1.
B. Interleukin 2.
C. Interleukin 3.
D. Interleukin 4.
E. Interleukin 6.
A

C - interleukin 3

IL1 - produced by macrophages, causes fever, cachexia, angiogenesis, activates immune cells
IL2 - produced by Th1 cells, causes proliferation and immune cell activation
IL4 - p/b Th2 cells and mast cells, B cell class switching to IgE, proliferation
IL6 - p/b T cells and macrophages, T and B cell proliferation and differentiation, fever

154
Q

A 15-year-old girl is brought into the emergency room by friends after attending a birthday party. Her
friends are concerned because there has been an alteration in her conscious state. She is febrile and
is visibly flushed and sweating. Her heart rate is 120/minute and regular. Her blood pressure is
160/95 mmHg. Her neurological exam was notable for her being combative with small but reactive
pupils. During your examination she begins to have a generalised seizure.
She has a history of depression and is being managed by a psychiatrist.
You suspect an ingestion or intoxication.
Which of the following is the most likely cause?
A. Alcohol.
B. Ecstasy.
C. Marijuana.
D. Mushroom intoxication.
E. Tricyclic antidepressants.

A

B - ecstasy

?Wrong, ecstasy causes: agitated, flushed, sweaty, DILATED pupils

?Depression managed by psychiatrist referring to TCA? 2004B paper. TCA overdose: convulsions, coma, prolonged QRS, DILATED pupils

Constricted pupils: opiates, organophosphates, clonidine
Dilated: amphetamines, atropine, TCA, antihistamine

155
Q

A two-year-old boy from Nigeria immigrated a year ago. He presents to the emergency department
with a fever of 40°C. His mother says that he has known sickle cell disease diagnosed previously in
Africa. Initial examination reveals a sick child along with an initial full blood count showing a raised
neutrophil count with left shift.
The most likely organism causing his fever is:
A. Epstein-Barr virus.
B. Haemophilus influenzae type b.
C. Plasmodium falciparum.
D. Salmonella typhi.
E. Streptococcus pneumoniae.

A

E - strep pneumoniae

Susceptibility is to pneumococci, Haemophilus, and Salmonella, due to multiple causes e.g. splenic dysfunction, defective complement activation, tissue ischaemia, micronutrient deficiency.

?Due to background, need to assume not vaccinated against Strep pneumo and Hib, and also not on prophylaxis, so Strep is most likely?

156
Q

A three-year-old girl is found floating face down in the family pool after being ‘missed’ for
approximately five minutes. Basic life support is commenced on the scene and on arrival in the
emergency department the girl is in asystole. She is apnoeic and pulseless, her temperature is 33°C
and the maximum estimated period of pulseless apnoea is 15 minutes. The child is rapidly intubated
and ventilated, intravenous access is gained and external cardiac compressions are continued.
Which of the following is the most appropriate next step in the acute management of this child?
A. Administer intravenous adrenaline.
B. Administer intravenous atropine.
C. Commence active warming measures.
D. Counsel the family and cease active resuscitative measures.
E. Immediate direct current (DC) defibrillation.

A

A - IV adrenaline

Follow the algorithm

157
Q

A 41-week gestation female infant was born by normal vaginal delivery with a birth weight of 3045 g
and head circumference of 33.5 cm. A rash on her limbs was noted on the first day of life. She
established breastfeeding very well. At 5 days of age, she was noted to be irritable and feeding
poorly. She developed left sided clonic jerks with eye deviation. These were controlled with
intravenous phenobarbitone.

The rash as illustrated above was described as an erythematous vesicular rash which was crusted in
places. It was present on the forearms and lower limbs.
The most likely cause of this rash would be:
A. congenital bullous ichthyosiform erythroderma.
B. congenital varicella.
C. epidermolysis bullosa.
D. herpes simplex virus infection.
E. incontinentia pigmenti.

A

E - incontinentia pigmenti.

X linked dominant, usually lethal in males.

Initially erythematous, papuler, vescular or bullous lesions on trunk and limbs, then pustular lesions then pigmented lesions.

Vesicular lesions in first 48 hours, verrucous lesions, streaky pigmentation and then atrophic pale lesions.
Associated with other abnormalities: skeletal, eye (30%), CNS (30-50%, seizures, encephalopathy), and dentition (80%).
Gene: NEMO

158
Q

A seven-year-old boy wakes in the morning and cries out to his parents to ‘come quickly’. He
complains to his parents that the ‘room is spinning’. The symptoms seem to be aggravated by head
motion and change of position from lying down. He becomes pale, sweaty and nauseated whenever
he moves or tries to sit up in bed. He is helped to the bathroom because he has difficulty standing
and walking unsupported and vomits repeatedly. His parents bring him to the emergency department
soon afterwards.

On examination, he is found to be perfectly conscious and cooperative if left to lie still in bed. He has
left-beating nystagmus in all positions of gaze but otherwise no focal neurologic findings. His hearing
is normal. He has a mild upper respiratory tract infection.
The most likely diagnosis is:
A. acute disseminated encephalomyelitis (ADEM).
B. acute labyrinthitis.
C. acute vestibular neuritis.
D. cerebellar ataxia.
E. Menière’s disease.

A

C - acute vestibular neuritis. Normal hearing = neuronitis, hearing loss = labyrinthitis.

Vestibular neuritis — Vestibular neuritis, also known as vestibular neuronitis and labyrinthitis, is believed to be a viral or postviral inflammatory disorder, affecting the vestibular portion of the eighth cranial nerve.

Vestibular neuritis is characterized by the rapid onset of severe, persistent vertigo, nausea, vomiting, and gait instability. Physical examination findings are consistent with an acute peripheral vestibular imbalance: spontaneous vestibular nystagmus, a positive head impulse (or head thrust) test, and gait instability without a loss of the ability to ambulate. In pure vestibular neuritis, auditory function is preserved; when this syndrome is combined with unilateral hearing loss, it is called labyrinthitis.

A diagnosis of vestibular neuritis is usually based on clinical information. The clinical features of cerebellar hemorrhage or infarction may be similar to vestibular neuritis, and brain imaging is therefore often required to rule this out.

Patients with vestibular neuritis generally suffer from severe vestibular symptoms for a few to several days, followed by a gradual diminution of symptoms and a return of equilibrium. Recovery may be improved with corticosteroid therapy.

159
Q

A couple who are first cousins are concerned about the risk of birth defects in their future children.
There is no known family history of genetic disease.
Their risk of having a baby with a birth defect is closest to:
A. 1%.
B. 2.5%.
C. 5%.
D. 10%.
E. 25%.

A

C - 5%

Google: Birth defects are common. Between 2% and 3% of infants have one or more defects at birth. That number increases to 5% by age one (not all defects are discovered directly after your child’s birth). One out of every 33 babies born in the United States are affected by birth defects.

160
Q

A 37-week gestation infant is noted to develop central cyanosis shortly after oral feeds on the third day
of life. Examination reveals a normal baby with no evidence of cyanosis or tachypnoea. A chest X-ray
shows a normal sized heart and clear lung fields.
The most likely diagnosis is:
A. congenital cyanotic heart disease.
B. congenital laryngeal cleft.
C. gastro-oesophageal reflux.
D. seizures.
E. tracheo-oesophageal fistula.

A

C - GOR. Much more common than TOF (this could really only be H type and that’s not common).

MRCPCH

Occurs as a result of the failure of development of the primitive foregut. Incidence ~1 in 3000.
85% blind proximal oesophageal pouch with a distal oesophageal to tracheal fistula, 10% with oesophageal atresia and no fistula, 5% with proximal +/- distal fistula

Clinical:

  • polyhydramnios
  • excessive salivation
  • early resp distress
  • abdominal distension
  • vomiting/choking on feeds
  • inability to pass NGT
  • absence of gast in gut if no fistula
  • other anomalies (VACTERL)
  • prematuriy common

H-type

  • much less common
  • usually not a/w preterm birth or other severe anomalies
  • may have delayed presentation with resp distress a/w feeding or recurrent LRTIs
161
Q

A two-year-old girl is referred to you for assessment of possible language delay. She has
approximately 30 single words in her vocabulary, but no two-word combinations. She has bilateral
glue ear (otitis media with effusion), and a recent hearing test showed a 30dB HL (hearing loss) pure
tone average in the left ear and a 15dB HL pure tone average in the right ear. Physical examination is
otherwise normal. She is too clingy for formal developmental assessment but her non-language
developmental milestones, as reported by her mother, seem normal.
What would be the best estimate of the chance of language delay at school entry?
A. 10%.
B. 33%.
C. 50%.
D. 67%.
E. 90%.

A

B - 33%

162
Q

A seven-year-old girl presents with a history of bleeding gums after she cleans her teeth. Examination
of her mouth reveals evidence of gingivitis as shown below.

A defect of which component of immune function is most likely to be associated with this clinical
presentation?
A. B cells.
B. Complement.
C. Natural killer cells.
D. Neutrophils.
E. T cells.
A

D - neutrophils

Unclear to me what the diagnosis is. Cyclic neutropenia p/w (cyclic pattern) stomatitis, oral ulcers, bacterial infections. LAD p/w (among other things e.g. delayed umbi separation) periodontitis and recurrent orogenital infections.

163
Q

A four-year-old boy with severe haemophilia A and a high titre factor VIII inhibitor needs surgery. The
peri-operative replacement coagulation product of choice is:
A. cryoprecipitate.
B. fresh frozen plasma.
C. recombinant activated factor VII.
D. recombinant factor VIII.
E. recombinant factor IX.

A

C - recombinant activated factor 7. Essentially in the setting of a high titre of inhibitor, unlikely to be overcome by increased dose of the normal factor, therefore need a “bypass product”, e.g. activated factor 7 (means the clotting cascade can be activated without the missing factors).

Uptodate

Overview of patient with an inhibitor — Management of bleeding or surgery in a patient with an inhibitor (neutralizing alloantibody against infused factor) is especially challenging because inhibitors bind to the infused factor and render it ineffective. Inhibitors are much more likely to occur in individuals with hemophilia A and those with severe disease. Some patients may only become aware that they have an inhibitor in the midst of an acute bleeding episode or surgery, when factor infusions no longer raise factor activity levels to the expected range or if there is an allergic/anaphylactic reaction to the infusion.

Inhibitors are diagnosed and classified by titer using the Bethesda assay, in which serial dilution of patient plasma is used to determine an inhibitor titer in Bethesda units (BU). Inhibitors with a titer of <5 BU despite repeated factor infusions are referred to as low-responding inhibitors. Any inhibitor ≥5 BU/mL at any time is considered high responding, even if the titer subsequently becomes undetectable due to lack of re-exposure. Classically, high-responding inhibitors rapidly increase upon re-exposure to infused factor in an anamnestic response that takes four to seven days.

Individuals with an inhibitor titer of ≥5 BU and those with an unknown titer but with a known high-responding inhibitor are less likely to be effectively treated with factor infusions in an emergency situation because the quantity of circulating antibody is likely too great to be overcome by factor infusion.
●For patients with a high-responding inhibitor whose current titer is <5 BU, it may be possible in some circumstances to treat them with factor coverage; this decision should only be made in consultation with a comprehensive hemophilia treatment center as anamnesis is expected.
●For a patient with a titer ≥5 BU with a high-responding inhibitor and serious bleeding with a need for major surgery, a bypassing product is used.

Bypassing products (rFVIIa products or FEIBA) — A bypassing product is generally the first choice in a patient with hemophilia A or B who has a high titer inhibitor and requires treatment for bleeding or surgery. These clotting factor products contain an activated form of a clotting factor in the coagulation cascade. Activated factor VII (factor VIIa) can directly activate factor X, bypassing the need for factors VIII and IX.

164
Q

An 11-year-old boy presents to the emergency department following a fall onto his outstretched left
arm whilst rollerblading. On examination his left hand is neurovascularly intact and there is no
deformity. His X-ray is shown below.

(Buckle radius fracture / nondisplaced radius fracture at wrist)

Which of the following is the most appropriate next step in management?
A. Apply backslab and review in one week.
B. Closed reduction in the emergency department.
C. Closed reduction in the operating theatre.
D. Open reduction.
E. Urgent orthopaedic review in the emergency department.

A

A - backslab and review 1 week

RCH fracture guidelines:

Buckle injury:
Below-elbow fibreglass/plaster backslab or removable wrist splint for 3 weeks
Check that both cortices are intact on the anteroposterior (AP) and lateral x-ray.
Provide parent with buckle injury fact sheet.
No follow-up by GP or fracture clinic is required.
No follow-up x-ray is needed

Complete - undisplaced or minimally displaced fractures
Refer to acceptable angulations (5-10yrs <15 degrees)
Below-elbow cast for 6 weeks
Fracture clinic within 7 days with x-ray

165
Q

A term newborn infant is noted to be tachypnoeic from delivery. On examination on day 1 he is also
noted to have prominent pulses and an active precordium. His liver is palpable 4 cm below the costal
margin. No murmurs are audible. His arterial oxygen saturation is 98% in room air.
Which of the following is the most likely diagnosis?
A. Aneurysm of the vein of Galen.
B. Group B streptococcal sepsis.
C. Hypoplastic left heart.
D. Patent (persistent) ductus arteriosus.
E. Thyrotoxicosis.

A

A - aneurysm of the vein of Galen

Google:

A vein of Galen malformation (VOGM) is a type of rare blood vessel abnormality inside the brain. In VOGM, misshapen arteries in the brain connect directly with veins, instead of connecting with capillaries, which help slow blood flow. This causes a rush of high-pressure blood into the veins.

This extra pressure in the veins can cause a number of problems:

The rush of blood toward the heart and lungs forces the heart to work overtime to get blood to the rest of the body. This can lead to congestive heart failure in some infants.
Blood pressure in the arteries from the heart to the lungs may rise, causing a serious condition called pulmonary hypertension.
The high pressure in the veins can prevent the infant’s brain from draining adequately. This can lead to widespread brain injury and sometimes causes severe loss of tissue in the brain.
Some infants can develop hydrocephalus an enlarged head) if the VOGM blocks the normal flow of fluid in the brain.
If not diagnosed and treated early, VOGM can cause severe problems and may even be life threatening.

166
Q

A 12-month-old girl is referred by her general practitioner because of failure to thrive. Her weight was
on the 50th percentile at six months but had fallen to the 10th percentile at one year. She had a moist
cough at nine months which lasted 10 days. Neonatal immunoreactive trypsin screening was
negative.
On examination she has buttock and thigh wasting. A full blood examination reveals a mild
leucocytosis and her electrolyte profile demonstrates the following:
serum sodium 120 mmol/L [135-145]
serum potassium 2.0 mmol/L [3.5-5.0]
serum chloride 90 mmol/L [90-110]
The most appropriate next investigation is:
A. anti-endomysial antibody.
B. serum and urinary osmolality.
C. serum 17-OH progesterone.
D. sweat test.
E. urinary sodium and potassium.

A

D - sweat test

Antiendomysial antibody (old question) for coeliac, doesn’t explain the electrolytes.
Serum and urine osmolality ?for DI, but no hx polyuria/dipsia etc.
Serum 17 OH progesterone is screening for 21 hydroxylase deficiency, the most common cause of CAH. Too old.
Urinary sodium and potassium ?again because no urinary history to suggest renal issue.

167
Q

A seven-month-old girl plays with a 15-month-old boy who has a cough and fever. The next day the
boy develops a morbilliform rash. A blood sample shows he has IgM antibodies to measles. You see
the girl the following day, two days after the initial contact.
What is the most appropriate course of action for the girl?
A. Administer normal human immunoglobulin alone and defer measles-mumps-rubella (MMR)
immunisation until the usual recommended time.
B. Administer normal human immunoglobulin and MMR vaccine concurrently, but in different
sites.
C. Immunise with MMR vaccine immediately.
D. Reassure the parents it is unlikely that their daughter will develop measles because of
maternal antibodies.
E. Warn the parents that their daughter is likely to develop measles, but that it is too late to do
anything.

A

A - Ig and defer MMR until usual time. Old question and now incorrect. As per immunisationhandbook.health.gov.au:

Normal human immunoglobulin is indicated for any immunocompromised patient or any patient who has not received a measles vaccination and is younger than 6 months of age, within 72 hours of exposure (and only if the mother is incompletely vaccinated or seronegative).

For this patient:
Give MMR vaccine now, then repeat dose at 12 months of age or 4 weeks later (whichever is later).
Give MMRV vaccine at 18 months of age, with a minimum interval of 4 weeks after the previous dose of MMR vaccine.

MMR usually given at 12 months and 18 months.

168
Q

A 13-year-old girl presents to your office with a six-month history of daily headaches. The headaches
are present on waking and on occasions are associated with vomiting without nausea. She has had
intermittent double vision and transient episodes of loss of vision lasting seconds only. She is taking
tetracyclines for acne.
On examination, she is moderately obese, afebrile, and has no neurocutaneous stigmata. She has a
moderate amount of facial acne. Her blood pressure is 105/65 mmHg. Her extraocular movements
show a partial left sixth nerve palsy and she has gross papilloedema. There are no other abnormal
findings.
A T2-weighted magnetic resonance imaging (MRI) scan of her brain is shown below.

(Dilated ventricles)

The most likely diagnosis is:
A. aqueductal stenosis.
B. benign intracranial hypertension (BIH).
C. craniopharyngioma.
D. meningioma.
E. temporal lobe tumour.
A

A - acqueductal stenosis

BIH/IIH should have normal parenchyma and ventricles on MRI.

169
Q

Diagnosis of a major depressive disorder according to the Diagnostic and Statistical Manual, 4th
edition (DSM-IV) requires a number of symptoms to have been present nearly every day during the
same two-week period.
Which of the following symptoms is least helpful in the diagnosis of adolescent depression?
A. Anhedonia (feeling of loss of pleasure in all, or almost all, activities).
B. Depersonalisation (feeling of being detached from oneself).
C. Impaired concentration.
D. Irritable mood.
E. Marked change in appetite.

A

B - depersonalisation

170
Q

An 11-year-old boy is admitted to hospital because he has been lethargic for the past three to four
weeks. He appears to be thirsty all the time with recent weight loss. He has been getting up to pass
urine three times each night for the past two weeks. There is no history of drug ingestion or infection.
There is no past history of note. Clinical examination is unremarkable.
His initial investigations show the following:
serum:
sodium 135 mmol/L [135-146]
potassium 3.0 mmol/L [3.5-5.0]
urea 16.0 mmol/L [3.2-7.7]
creatinine 0.29 mmol/L [0.04-0.08]
glucose 3.5 mmol/L [2.5-7.0]
urinalysis 1+ protein, no blood, moderate glucose
urine microscopy 40 white cells, <10 red cells, no bacteria, no growth
The most likely diagnosis in this patient is:
A. acute glomerulonephritis.
B. acute interstitial nephritis.
C. acute tubular necrosis.
D. diabetes insipidus.
E. Goodpasture syndrome.

A

B - AIN. ?Fanconi syndrome (urine positive for protein, glucose, serum hypokkalaemia…).

Uptodate

Acute interstitial nephritis (AIN) is a renal lesion that typically causes a decline in kidney function and is characterized by an inflammatory infiltrate in the kidney interstitium [1]. It is most often induced by drug therapy. AIN is also caused by autoimmune disorders or other systemic disease (eg, systemic lupus erythematosus [SLE], Sjögren’s syndrome, sarcoidosis), a variety of infections remote to the kidney (eg, Legionella, leptospirosis, and streptococcal organisms), and tubulointerstitial nephritis with uveitis (TINU) syndrome.

he distribution of causes of AIN has been reported as follows:
●Drugs – 70 to 75 percent (with antibiotics responsible for approximately 30 to 50 percent of these cases)
●Systemic disease including sarcoidosis, Sjögren’s syndrome, systemic lupus erythematosus (SLE), and others – 10 to 20 percent
●Infections – 4 to 10 percent
●Tubulointerstitial nephritis and uveitis (TINU) syndrome – less than 5 percent

Classically, patients with drug-induced AIN were reported to have symptoms and/or signs of an allergic-type reaction, including rash, fever, and eosinophilia. However, in a more recent review of three series that totaled 128 patients with AIN (of whom 70 percent had drug-induced disease), these findings of a typical allergic response were relatively less common at presentation:
●Rash – 15 percent
●Fever – 27 percent
●Eosinophilia – 23 percent
●Triad of rash, fever, and eosinophilia – 10 percent

Signs of tubulointerstitial damage, such as the Fanconi syndrome and renal tubular acidosis, may be present but rarely dominate the clinical picture.

171
Q

A 14-year-old girl complains of tiredness. She has no significant past medical history and denies any
medications. Apart from being thin, there are no abnormal findings on examination. Her blood
pressure is 105/70 mmHg.
Results of investigations are listed below.

Serum biochemistry:
sodium 138 mmol/L [135-145]
potassium 2.3 mmol/L [3.4-5.0]
chloride 85 mmol/L [103-109]
creatinine 0.10 mmol/L [0.06-0.12]
Arterial blood gases:
pH 7.50 [7.34-7.45]
PaO2 95 mmHg [80-100]
PaCO2 42 mmHg [35-45]
bicarbonate 39 mmol/L [22-28]

Urinary biochemistry:
sodium 30 mmol/L
potassium 42 mmol/L
chloride 13 mmol/L

The most likely explanation for these results is:
A. Bartter syndrome.
B. occult diuretic use.
C. occult laxative use.
D. primary hyperaldosteronism.
E. self-induced vomiting.
A

E - self induced vomiting (like hypertrophic pyloric stenosis, hypokalaemic hypochloraemic metabolic alkalosis).

Bartter syndrome: D/T inborn autosomal recessive defect in sodium, potassium, chloride cotransporter in loop of Henle -> NaCl and water wasting. Polyuria, polydipsia, dehydration, faltering growth, constipation. RAAS activation retains Na at the expense of K and H. Leads to hypochloraemic hypokalaemic alkalosis. Urine chloride and sodium are inappropriately high (>20).

172
Q

An 18-month-old child presents after swallowing a 20-cent piece. At which of the following anatomical
sites is the foreign body most likely to become impacted?
A. Duodeno-jejunal junction.
B. Gastro-oesophageal junction.
C. Ileocaecal valve.
D. Oesophagus at level of cricoid.
E. Pylorus.

A

D - oesophagus at the cricoid.

173
Q

A four-year-old boy presents with worsening back pain, which is more severe at night. The pain first
started about three weeks previously after a minor upper respiratory infection and was initially
intermittent. There has not been any recent history of fever. He had previously been well, with no
significant infections other than otitis media.
Examination reveals tenderness over the lumbar spine at L2/3, limited straight leg raising and a loss of
the normal lumbar lordosis. His erythrocyte sedimentation rate (ESR) is 60 mm/hr and his C-reactive
protein is 4 mg/L [<8].
A magnetic resonance imaging (MRI) scan of his spine is shown below.

The most likely diagnosis is:
A. ankylosing spondylitis.
B. discitis.
C. epidural abscess.
D. osteoid osteoma.
E. tumour.
A

B - discitis. 2004B.

174
Q
The most common adverse effect of methylphenidate is:
A. appetite suppression.
B. headache.
C. sleep difficulty.
D. tics.
E. withdrawn behaviour.
A

A - appetite suppression

175
Q

Antenatal maternal smoking is least associated with which one of the following effects in children?
A. Decreased lung compliance.
B. Decreased maximal expiratory flow.
C. Increased risk of lower respiratory tract infections.
D. Increased risk of sudden infant death syndrome.
E. Increased risk of wheezing.

A

A - decreased lung compliance

176
Q
A 10-year-old girl with a longstanding history of migraine, on no medications, presents with severe
headache and left upper limb weakness.
Which of the following medications should not be used at this point?
A. Codeine.
B. Cyproheptadine.
C. Dihydroergotamine.
D. Ibuprofen.
E. Propranolol.
A

C - dihydroergotamine (contraindicated in hemiplegia migraines).

177
Q

A five-year-old girl presents with a two-day history of fever to 38°C axillary and a swollen, tender right
knee. She has previously been well, and has had no rash and no previous episodes of joint pain. On
examination, she is febrile (37.9°C) and flushed. She has a heart rate of 100/minute with normal heart
sounds. She has shotty cervical lymphadenopathy, but no palpable liver or spleen. Her right knee is
warm, swollen, red, and slightly tender. She has no effusion clinically. She can straighten her leg and
flex the right knee, but there is pain at the extreme of knee flexion.

Her blood test results are as follows:
haemoglobin 112 g/L [110-140]
white cell count 10.8 x 109/L [4.0-11.0]
differential:
neutrophils 3.5 x 109/L [1.0-4.0]
lymphocytes 7.3 x 109/L [3.0-8.0]
platelet count 170 x 109/L [150-400]

erythrocyte sedimentation rate (ESR) 18 mm/hr [<10]
Which of the following is the most likely diagnosis?
A. Acute leukaemia.
B. HLA B27-associated arthritis.
C. Pauci-articular juvenile idiopathic arthritis.
D. Septic arthritis.
E. Viral arthritis.

A

E - viral arthritis.

White cell differential goes against bacterial, as does lymphadenopathy, and symptoms are relatively mild (should be extremely painful to move a septic joint).

178
Q
An active six-year-old boy has a clear cut school refusal problem. Which of the following is most likely
to be effective?
A. Change in school.
B. Family therapy.
C. Gradual exposure to school.
D. Parental presence in the classroom.
E. Sertraline.
A

C - gradual exposure to school

179
Q

A six-year-old boy presents with a sore throat, high fever and anorexia. The appearance of his mouth
and tongue is shown below.

The most likely organism responsible for this picture is:
A. Candida albicans.
B. Coxsackievirus A16.
C. Epstein-Barr virus.
D. Fusobacterium necrophorum.
E. herpes simplex virus.
A

E - HSV.

Too old for HFM.

180
Q
A child presents with a sore throat. Which of the following clinical features is least likely in Group A
streptococcal tonsillitis?
A. Age 10 years.
B. Cough.
C. Exudate.
D. Fever >39°C.
E. Tender enlarged cervical lymph nodes.
A

B - cough

Uptodate

Group A Streptococcus (GAS), also known as Streptococcus pyogenes, is the most common cause of bacterial pharyngitis in children and adolescents. GAS is a facultative, gram-positive coccus that grows in chains. The only known reservoirs are the skin and mucous membranes of the human host.

GAS is the most common cause of bacterial pharyngitis in children and adolescents. It accounts for 15 to 30 percent of all cases of pharyngitis in children between the ages of 5 and 15 years.

Children ≥3 years — In children ≥3 years, GAS pharyngitis typically has an abrupt onset. Fever, headache, abdominal pain, nausea, and vomiting may accompany the sore throat, which can lead to poor oral intake. Additional features may include exudative tonsillopharyngitis, with enlarged erythematous tonsils, enlarged tender anterior cervical lymph nodes, palatal petechiae, inflamed uvula (uvulitis), and scarlatiniform rash (erythematous, finely papular rash which characteristically starts in the groin and axilla and then spreads to the trunk and extremities, followed by desquamation). Symptoms usually resolve spontaneously in three to five days.

181
Q
A five-month-old boy presents with chronic diarrhoea, failure to thrive and recent onset cough
associated with feeding intolerance. Blood test results show:
haemoglobin 98 g/L [110-150]
white cell count 14 x 109/L [4-11]
differential:
neutrophils 11 x 109/L [1-7]
lymphocytes 3 x 109/L [1-4]
platelet count 90 x 109/L [150-400]

IgG 9.0 g/L [3.3-6.4]
IgA 0.67 g/L [0.30-0.70]
IgM 1.90 g/L [0.25-0.60]
A chest X-ray was taken and is shown below.

Which one of the following is the most likely cause of this clinical presentation?
A. Human immunodeficiency virus (HIV) infection.
B. Hyper IgM syndrome.
C. Severe combined immunodeficiency.
D. Wiskott-Aldrich syndrome.
E. X-linked agammaglobulinaemia.

A

A - HIV. As a learning point, HIV should be excluded prior to the diagnosis of SCID (similar presentation of faltering growth and spectrum of infectious pathogens).

Hyper IgM - IgA should be low due to inability to class switch.
SCID - lymphocytes should be low. Lymphopenia and hypogammaglobulinaemia.
Wiskott Adrich - PID, eczema, thrombocytopenia.
X-linked agammaglobulinaemia - no immunoglobulin, IgM and IgA should be low.